Interesting Cases

Metformin Overdose and Lactic Acidosis

Lactic Acidosis is a common cause of metabolic acidosis.  Metabolic acidosis has several effects on human physiology, the most serious of which is the lowering of the ventricular fibrillation threshold of the heart.  Lactic acidosis in popular literature is connected to excessive exercise, but there are other causes for lactic acidosis including dehydration, starvation, severe anemia, diabetic ketoacidosis, and other conditions.  The paper that follows is a case study published in a 2012 issue of J. Med having to do with an overdose of Metformin, an oral hypoglycemic agent used to treat Type II diabetics.  It is a very interesting toxicology case.

Read more: Metformin Overdose and Lactic Acidosis

Antidotes for Toxicologic Emergencies

In toxicology, true antidotes are scarce.  The following article addresses some of the mainstay antidotes in several toxicologic poisonings.  It is important for clinicians to recognize these conditions quickly because the efficacy of these antidotes are time-dependent.

Introduction

Poisoning is a leading cause of morbidity and mortality in the United States;[1] in fact, it is the second leading cause of injury-related mortality, and its incidence is rising. The American Association of Poison Control Centers' National Poison Data System receives reports of more than 2.4 million human poison exposures and approximately 1300 poisoning-related deaths annually.[2] However, it is likely that the associated mortality is much higher than those statistics would indicate, as it is estimated that only about 5% of U.S. poisoning deaths are reported to poison control centers.[3,4]

Antidotes play a critical role in the care of poisoned or overdosed patients. Recently issued national consensus guidelines include a recommended list and the quantities of antidotes that should be readily available in hospitals that provide emergency care.[5] Some of the anti-dotes should be available for immediate administration on a patient's arrival, which requires stocking in the emergency department (ED) at most hospitals; other antidotes should be available within 60 minutes and can be stocked in the hospital pharmacy provided that prompt delivery to the ED can be assured. A recommended antidote stocking list and sample inventory log can be found in eFigure 1 and eTable 1, available at www.ajhp.org. This list should be adapted by each individual facility based on a needs assessment. Most importantly, this list should be decided on by vested parties (e.g., pharmacists, physicians, and other health care practitioners involved in providing emergency care and critical care).

One of the many roles of the ED pharmacist is participating in the management of toxicological emergencies. The goal of this review is to provide essential information to guide the appropriate use of antidotes. The antidotes discussed (in alphabetical order) are those whose use likely entails the greatest involvement of ED pharmacists. As recommendations may change, clinicians should always consult a regional poison control center (1-800-222-1222) to ascertain the most current recommendations on antidote use and to report exposures and poisonings.


Antidotes for Toxic alcohol-analog Poisoning

The use of ethanol or, preferably, fomepizole for alcohol dehydrogenase (ADH) inhibition is a mainstay in the management of toxicity due to ingestion of methanol, ethylene glycol, or diethylene glycol.[6–8]

The toxicity of methanol and of ethylene glycol is well described, and each year in the United States there are about 5000 exposures that require treatment and 20–30 associated deaths reported to poison centers.[2,9,10] Methanol and ethylene glycol, as parent compounds, are relatively nontoxic. However, they are metabolized by ADH to toxic metabolites that can cause end-organ damage and death. Methanol is metabolized via ADH to formic acid, which results in anion-gap metabolic acidosis and ocular toxicity. Retinal toxicity secondary to methanol poisoning is usually irreversible.[6,11] Ethylene glycol is metabolized via ADH to glycolic acid, which results in anion-gap metabolic acidosis, and oxalic acid, which results primarily in renal toxicity due to the formation of calcium oxalate crystals.[7,12] Both can produce irreversible CNS toxicity.

Poisoning by diethylene glycol (historically and tragically used as a glycerin substitute and also in household products such as wallpaper stripper and Sterno brand heating fuel[13,14]) is less common but associated with very high morbidity and mortality.[15,16] Diethylene glycol is metabolized via ADH to hydroxy-ethoxyacetic acid and diglycolic acid and causes anion-gap metabolic acidosis, bilateral cortical necrosis, and sensorimotor polyneuropathy.[16–20]

Ethanol

For many years, ethanol has been used to inhibit ADH and limit the metabolism of methanol and ethylene glycol to their respective metabolites.[21] The dose of ethanol needed to competitively inhibit ADH depends on the comparative affinity of the specific toxic alcohol for ADH. Most authorities recommend using a dose of ethanol sufficient to achieve and maintain a serum ethanol concentration of 100–150 mg/dL. In the presence of ethanol, the half-lives of ethylene glycol (in patients with normal renal function) and methanol are approximately 17.5 and 45 hours, respectively.[6,7]

Ethanol can be administered intravenously or orally. However, a commercial i.v. preparation of ethanol is no longer available, and extemporaneous preparation is too time-consuming to be considered satisfactory. A loading dose is necessary to quickly achieve the desired serum concentration of 100–150 mg/dL; then a maintenance dose is administered, using serum ethanol concentrations to maintain the desired target. Repeat evaluations of the serum ethanol concentration are required to ensure that the target level is achieved and maintained. Individual differences in ethanol metabolism occur due to pharmacogenetics and whether the patient is induced or becomes induced secondary to chronic ethanol exposure.[6,7]

The risks associated with ethanol administration include central nervous system (CNS) depression, hypoglycemia (due to decreased gluconeogenesis), nausea, and vomiting. Intravenous administration of ethanol poses an additional risk of phlebitis and hypertonicity with hyponatremia. Frequent assessment of the serum ethanol concentration and monitoring of venous blood glucose are required.

Fomepizole

Fomepizole competitively inhibits ADH and is an effective and safe antidote for both ethylene glycol and methanol toxicity.[6,7] In the presence of fomepizole, the half-lives of ethylene glycol (in patients with normal renal function) and methanol are 14.5 and 40 hours, respectively.[22]

The Food and Drug Administration (FDA)-approved regimen of fomepizole is an i.v. loading dose of 15 mg/kg over 30 minutes followed by a dose of 10 mg/kg every 12 hours, with the frequency of dosing increased to every 4 hours during hemodialysis.[23] Fomepizole induces its own metabolism, presumably through the cytochrome P-450 2E1 isoenzyme; therefore, after 48 hours of drug administration, the fomepizole dose should be increased to 15 mg/kg every 12 hours.

Fomepizole is generally well tolerated. Adverse events reported with the use of fomepizole include mild irritation at the i.v. infusion site, headache, nausea, dizziness, drowsiness, and a bad or metallic taste in the mouth.

Although there are no head-to-head comparisons of fomepizole versus ethanol for the management of toxic-alcohol poisoning, the former's ease of administration and relative lack of serious adverse effects have elevated it to preferred status. The clinical advantages of fomepizole over ethanol are a much higher potency of ADH inhibition (K i = 0.1μmol/L, a

1000-fold higher affinity than that of ethanol), better maintenance of therapeutic blood concentrations, and fewer adverse effects; moreover, the administration of fomepizole is less labor-intensive.[12]

Additional and Supportive Therapy

In addition to antidote administration, hemodialysis should be considered in all toxic-alcohol exposures in which toxic metabolites have already formed, as evidenced by anion-gap metabolic acidosis or end-organ damage, and for patients with toxic serum methanol or ethylene glycol concentrations whose elimination of parent or toxic metabolites is expected to be inordinately prolonged (e.g., cases involving significant methanol exposure or ethylene glycol ingestion by a patient with renal impairment). Empiric hemodialysis is recommended if the serum methanol concentration is >25 mg/dL and if the serum ethylene glycol concentration is >50 mg/dL with renal insufficiency.[6,7] Hemodialysis also should be considered in cases of severe isopropyl alcohol poisoning in patients with hemodynamic instability.

Intravenous administration of 50 mg of folic acid every six hours enhances methanol elimination and has been shown to prevent retinal toxicity in animal models.[8,24,25] Also, urinary alkalinization (i.e., a urine pH of >8) with i.v. sodium bicarbonate enhances formate elimination and may reduce the distribution of formic acid to the eye.

Theoretically, the use of i.v. thiamine hydrochloride 100 mg and i.v. pyridoxine hydrochloride 50 mg every six hours should shunt the metabolism of ethylene glycol away from production of oxalic acid to production of less toxic metabolites'[26,27] though there are no data from studies of humans to support this practice, these agents are well tolerated and the potential benefits outweigh any risks.

Implications for the Pharmacist Methanol or ethylene glycol toxicity should be suspected in a patient with anion-gap metabolic acidosis in whom laboratory testing reveals a low (or no) ethanol concentration, no ketones, and a normal lactic acid concentration (clinicians need to be aware that some test results can be skewed by glycolic acid, the toxic metabolite of ethylene glycol). Fomepizole and adjuvants that act as cofactors should be used as soon as toxic alcohols are included in the differential diagnosis. Fomepizole should be continued until the patient is no longer acidemic and the toxic-alcohol serum concentration is presumed or confirmed to be <25 mg/dL. The availability of testing for toxic alcohols is limited.

Antidotes for Calcium-channel-blocker and β-blocker Toxicities

Toxicity due to calcium-channel blockers (CCBs) or β-blockers results in significant morbidity and mortality.[9] The manifestations of toxicity are generally extensions of the drugs' pharmacologic and therapeutic effects and often include hypotension, bradycardia, conduction block, and myocardial depression.[28,29] Depending on the amount of the offending drug ingested and the patient's underlying cardiovascular health, the patient could remain asymptomatic or progress to cardiovascular collapse.

Subtleties in presenting symptoms can help differentiate CCB and β-blocker poisoning. Patients experiencing a CCB overdose tend to remain awake and alert, even in the event of profound hypotension and bradycardia, while patients with β-blocker poisoning are more likely to have an altered mental status and respiratory depression.[29] The more severe the CCB overdose, the more likely the patient is to exhibit hyperglycemia, because CCBs also inhibit the release of insulin from pancreatic β-cells via a calcium-dependent pathway.[30] Children experiencing a β-blocker overdose may develop hypoglycemia, an uncommon symptom in adults. Dihydropyridine CCBs such as nifedipine are more potent peripheral vasodilators than nondihydropyridine CCBs; they have limited effects on cardiac rhythm and are more likely to cause hypotension with reflex tachycardia. Propranolol, a β-blocker with high lipophilicity and sodium-channel-blocking effects, is more likely than other β-blockers to cause patients to have a seizure and to exhibit a widened QRS complex on electrocardiography.[28,29] Toxicity resulting from the ingestion of the combination of a β-blocker and a CCB can be particularly serious and life-threatening. Even at therapeutic doses, the ingestion of more CCB or β-blocker medication than is prescribed can be life-threatening in a patient with a tenuous cardiac history.

Because of the pathophysiologic similarities of CCB toxicity and β-blocker toxicity, their management is similar. The treatment of patients with bradycardia and hypotension begins with fluids and atropine, but patients who are more than mildly poisoned typically do not have an adequate response to these therapies. Other treatment modalities include calcium, glucagon, hyperinsulinemia–euglycemia therapy (HIET), vasopressors, cardiac pacing, i.v. 20% fatty acid emulsion, extracorporeal circulatory support, and intra-aortic balloon pump therapy.

Calcium

Calcium plays an integral role in myocardial function and is necessary for automaticity, conduction, contraction, and vascular tone. In theory, the administration of exogenous calcium to patients with CCB toxicity should competitively increase calcium entry into the myocardium via nonblocked channels. Calcium has also been used to treat β-blocker toxicity.[28]

Along with atropine, calcium is considered a first-line therapy for CCB or β-blocker toxicity. Patients with mild toxicity seem to have an adequate response to calcium therapy; those with severe toxicity usually require additional therapies.

Calcium is available as either calcium chloride or calcium gluconate. Because of differences in the molecular weights of the chloride and gluconate components, 30 mL of 10% calcium gluconate is equivalent to 10 mL of 10% calcium chloride. Extravasation of calcium must be avoided. In particular, calcium chloride is extremely damaging to tissue should extravasation occur. For this reason, it is recommended that calcium chloride be administered through a central line or only with good peripheral venous access. Care should also be taken not to extravasate calcium gluconate, but the consequences are less severe, so the administration of calcium gluconate through a peripheral vein is more appropriate.

A reasonable starting dose in adults is 30 mL of calcium gluconate or 10 mL of calcium chloride, with additional doses administered in 15–20 minutes. After three doses, careful monitoring of ionized calcium is necessary to avoid dangerous hypercalcemia. Calcium is administered to improve hemodynamics.

Hypercalcemia may lead to an ileus, myocardial depression, hyporeflexia, and an altered mental status. The administration of calcium to a patient with cardioactive-steroid (e.g., digoxin) toxicity may lead to asystole and should be avoided.

Implications for the Pharmacist To avoid hypercalcemia and its associated risks, close monitoring of the serum ionized calcium level is required, especially in patients receiving multiple doses of exogenous calcium. The use of calcium should be avoided in a patient with known or suspected digoxin toxicity.

Glucagon

It is glucagon's ability to increase cardiac cyclic adenosine monophosphate (cAMP) directly and independently of the β- adrenergic receptor that has established its role in the management of β-blocker overdoses.[29] The increase in cardiac cAMP enhances inotropy and chronotropy and may improve conduction. Glucagon can also be used to manage CCB toxicity because not only is it difficult to distinguish an overdose of a β-blocker from an overdose of a CCB, as the two types of medications are frequently consumed together, but also because the glucagon-induced increase in cAMP occurs regardless of whether the calcium channel is blocked. In severely poisoned patients, glucagon will likely be ineffective and additional interventions are necessary.[29]

Glucagon causes dose-dependent and rate-related nausea and vomiting with a risk of aspiration; thus, antiemetics such as metoclopramide and serotonin antagonists are often used in patients receiving the drug.[29] Other adverse effects of glucagon can include hyperglycemia, followed by hypoglycemia in rare cases; gastrointestinal (GI) smooth-muscle relaxation and diarrhea; hypokalemia; and, rarely, allergic reactions. Tachyphylaxis with continued administration of glucagon is a theoretical concern.

Glucagon has a rapid onset of action and a short duration of effect, rarely longer than 15 minutes. As with other therapies used in toxicology, definitive glucagon dosing recommendations are lacking; a dosage of 50 μg/kg, or 3–5 mg up to a


cumulative dose of 10 mg, is reasonable.[28] This dose can be repeated as necessary. If there is a favorable response to glucagon boluses, a continuous infusion may be used.

Implications for the Pharmacist The doses of glucagon necessary for the management of β-blocker or CCB toxicity are much higher than those typically used to induce hyperglycemic or antispasmodic effects. The endpoints for discontinuing glucagon infusions are not clear; however, it is reasonable that once a patient is hemodynamically stable for a minimum of 6 hours, a slow taper of a single agent at a time can be employed. Anecdotal evidence and clinical experience suggest that once therapy is discontinued, close observation is necessary for a minimum of 12 hours.

HIET

The management and outcomes of patients severely poisoned by CCBs or β-blockers have improved substantially since the advent of HIET.[31,32] High-dose insulin has long been reported to be an inotrope.[33] It was only in the late 1990s that HIET was demonstrated to be effective in treating patients severely poisoned with CCBs or β-blockers.[32] The mechanism of HIET's effectiveness has not been clearly delineated; the available data suggest it enhances carbohydrate use and energy production by myocardial cells, resulting in improved contractility.[34–37] Because of the alterations in myocardial cell metabolism, it is not surprising that the beneficial effects of HIET in patients with CCB or β-blocker toxicity are delayed, generally occurring after 15–60 minutes.[37–39] Therefore, HIET should be started early in the course of management. If a patient remains hypotensive and bradycardic after receiving fluids, atropine, calcium, and glucagon, HIET should be administered. As HIET is particularly effective in improving myocardial contractility, the early administration of HIET may avoid the need for vasopressors or allow the use of lower doses, thereby reducing the potential for ischemic consequences.[39]

The major adverse effects associated with HIET are hypoglycemia and hypokalemia. The sicker the patient is from a CCB overdose, the more likely it is that hyperglycemia will develop before HIET is instituted; as the patient recovers, the need for supplemental glucose increases.[30] Insulin causes an intracellular shift of serum potassium, and potassium supplementation should be considered when the serum potassium concentration is <3 meq/L.

HIET should begin with an i.v. loading dose of 1 unit/kg of regular insulin followed by an infusion of 0.5–1 unit/kg/hr.[39] The infusion dosage can be increased every 20–30 minutes. Doses of 2.5–3 units/kg/hr have been used depending on the response. Experimental studies have used even higher doses.[40] Serum glucose should be maintained at a concentration of >100 mg/dL during HIET. A maximum insulin dose has not been established. If the initial blood glucose concentration is <400 mg/dL, an i.v. loading dose of 0.5 g/kg dextrose should be administered with the insulin and followed by an infusion of 0.5 g/kg/hr of dextrose, with meticulous and frequent monitoring of serum glucose and potassium concentrations.[39] This dose of dextrose can be administered in a concentrated form (e.g., a 20–25% concentration) through a central line to avoid problems with fluid overload and venous irritation. The recommended goal is to maintain a serum glucose concentration of 100–250 mg/dL. A patient with a falling glucose concentration should be treated by increasing the amount of supplemental glucose (not by decreasing the insulin infusion) until the patient is hemodynamically stable.

Implications for the Pharmacist The use of HIET has resulted in a decline in mortality among patients with severe CCB or β-blocker toxicity. There is a delay in the benefits of HIET, so it should be started early. A general rule of thumb is to initiate HIET when it is apparent that calcium and glucagon are ineffective or as soon as the decision is made to initiate a vasopressor.

Antidotes for Cyanide Poisoning

In the United States, there are now two types of cyanide antidotes available. The Lilly Cyanide Antidote Kit was the first and, for many years, the only such kit available; it contained amyl nitrite, sodium nitrite, and sodium thiosulfate. This combination of agents is now available as the branded Cyanide Antidote Package or as the generic Cyanide Antidote Kit, and the components are sold separately by various manufacturers.[41] Nithiodote, recently approved by FDA, contains sodium nitrite and sodium thiosulfate.[42] In 2006, FDA approved hydroxocobalamin, a novel cyanide antidote, available as the branded Cyanokit.[43]

Cyanide binds to the ferric ion on cytochrome oxidase and abruptly halts the electron transport chain and aerobic respiration, producing profound toxic effects. Cyanide also preferentially binds to the ferric ion of methemoglobin, but endogenous concentrations of methemoglobin are quite low.

Exposure to cyanide can occur during house fires, industrial accidents, and attempted suicides, and cyanide is a potential agent of chemical warfare. Smoke inhalation is one of the more common sources of exposure to cyanide in the United States.[44,45] Suicide attempts involving the ingestion of commercially available cyanide salts have been reported.[46] Cyanide overdose has been reported among workers in the gold, jewelry, and textile industries, in which the salts are frequently used. Cyanide exposure causes rapid, severe systemic toxicity and rapid cardiovascular collapse. Although there is no rapid test for the diagnosis of cyanide poisoning, an elevated lactate concentration (>8 mmol/L, or 72 mg/dL)[47] and a venous blood gas with a high partial pressure of oxygen and a high oxygen saturation are, in the appropriate clinical context, highly suggestive of cyanide toxicity and warrant empiric antidotal therapy.

Amyl Nitrite and Sodium Nitrite

The mechanism of action of amyl nitrite and sodium nitrite as antidotes for cyanide poisoning is to produce methemoglobinemia and vasodilation.[48] Vasodilation may contribute to their therapeutic and adverse effects.

Intravenous sodium nitrite produces significant methemoglobinemia.[49] The cyanide bound to cytochrome oxidase is then preferentially bound to methemoglobin, forming cyanomethemoglobin. Rhodanese, an endogenous enzyme, then facilitates the formation of thiocyanate, a much less toxic metabolite, which is renally excreted.

The creation of methemoglobinemia through the use of nitrites for cyanide poisoning entails some risk and, in particular, may be detrimental or even lethal to a patient with smoke inhalation and concurrent carboxyhemoglobinemia or lung injury.[50] Neither carboxyhemoglobin nor methemoglobin is capable of carrying oxygen, so such patients can develop functional hypoxia. Therefore, the administration of the nitrite component of therapy for cyanide poisoning should be avoided in patients with smoke inhalation unless it can be demonstrated that the carboxyhemoglobin level is negligible. The dosage of nitrites should not be adjusted to achieve a predetermined methemoglobin concentration, since the formation of cyanomethemoglobin can potentially be misread as methemoglobin formation by an oximeter during patient monitoring. (A methemoglobin concentration above 20% should halt further nitrite administration.)

In the context of cyanide poisoning, the differences between nitrites lie in the route of administration and the degree of methemoglobinemia they produce.[49,51] Amyl nitrite is inhaled, produces a minimal amount of methemoglobin, and is designed to be administered pending the establishment of i.v. access, as is often the case in the prehospital setting. Sodium nitrite is administered intravenously and results in a methemoglobin concentration of about 15% in healthy adults.[49] In other scenarios of cyanide toxicity, particularly the intentional ingestion of cyanide salts, amyl nitrite can be given to adults as one ampul (0.3 mL) inhaled until i.v. access is obtained, followed by 300 mg (10 mL of 3%) i.v. sodium nitrite over two to four minutes.[41] Children should receive 6 mg/kg (0.2 mL/kg of 3%) sodium nitrite up to the adult dose, at the same rate. This dosing strategy has been established as safe in children with a hemoglobin concentration of ≥7 g/dL). Half of the recommended dosage can be administered if cyanide toxicity reappears or, for prophylaxis, two hours after the initial dosage.

Sodium Thiosulfate

As noted above, cyanide is metabolized by the enzyme rhodanese to a less toxic metabolite, thiocyanate, which is renally eliminated. However, this metabolic pathway is capacity limited. Thiosulfate enhances the activity of rhodanese by donating a sulfur group, thereby increasing the amount of thiocyanate that rhodanese can produce.[52] Sodium thiosulfate is relatively well tolerated, but there is a potential for nausea and vomiting, as well as rate-related hypotension.

Because of its relatively favorable adverse-effect profile, sodium thiosulfate should be given to all patients with suspected cyanide toxicity, including those with smoke inhalation. The recommended dosage of sodium thiosulfate for adults is 12.5 g i.v. (50 mL of 25% solution); for pediatric patients, it is 0.5 g/kg i.v. (2 mL/kg of 25% solution) up to the adult dose. One half of the initial dose can be administered two hours later if toxicity reappears or as a preventive measure. Intravenous

sodium thiosulfate should be administered either as a bolus injection or infused over 10–30 minutes immediately after sodium nitrite via the same i.v. line.[41]

Hydroxocobalamin

Vitamin B12a, or hydroxocobalamin, detoxifies cyanide and forms cyanocobalamin, which is renally excreted.

Hydroxocobalamin is an appealing cyanide antidote because it is relatively safe, does not compromise the blood's oxygen- carrying capacity, and, unlike the nitrites or sodium thiosulfate, does not produce hypotension. These features make hydroxocobalamin an ideal agent for empiric use in patients with smoke inhalation who are suspected to have cyanide toxicity.[53,54] Hydroxocobalamin has been found effective for the treatment of acute cyanide poisoning in animal models; in one study of laboratory dogs rendered cyanide toxic, mortality was greatly reduced among dogs given hydroxocobalamin in comparison with those given placebo (21% versus 82%, respectively).[55]

In healthy volunteers, the use of hydroxocobalamin has been linked to chromaturia, dose-dependent erythema, headache, GI distress, pruritus, dysphagia, and infusion-site reactions. Allergic reactions are less frequent but occasionally are severe enough to require intervention. In one study, 25% of volunteers who received hydroxocobalamin experienced a substantial rise in diastolic blood pressure, and three also had a rise in systolic blood pressure; these blood pressure changes were attributed to the effects of hydroxocobalamin on nitric oxide scavenging.[56] A delayed pustular rash appeared on the face and neck of a few participants in that study and took several weeks to resolve.

Hydroxocobalamin is known to cause a reddish discoloration of the urine that typically resolves within 48 hours.[57] Once hydroxocobalamin is administered, the use of laboratory tests that depend on colorimetric techniques is no longer valid, as both hydroxocobalamin and cyanocobalamin are bright red and will cause interference; assays for bilirubin, creatinine, aspartate transaminase (AST), iron, glucose, magnesium, and hemoglobin and most urine assays are among the tests affected. There was a recent case report of a hydroxocobalamin-related colorimetric change resulting in problems with hemodialysis; the machine interpreted the discoloration as a blood leak and shut down automatically.[58] Discoloration of urine by hydroxocobalamin also has been reported to interfere with a spectroscopic assay for urinary thiocyanate that is often used to confirm cyanide exposure.

The use of hydroxocobalamin can also skew the results of serum carboxyhemoglobin determinations, falsely increasing or falsely decreasing the measured concentration. Therefore, if possible, blood samples should be drawn before the administration of hydroxocobalamin.[59–62]

The empiric adult dose of hydroxocobalamin is 5 g, which can be infused over a period of 15 minutes, with the infusion repeated if necessary; the pediatric dose is 70 mg/kg, up to a maximum of the adult dose, administered at the same infusion rate.[43]

There are no published data on the compatibility of hydroxocobalamin with other substances, and the drug should therefore not be administered through the same line as other agents.[43] If another line is not available, sodium thiosulfate can be administered through the same line after hydroxocobalamin administration is completed, with care taken to avoid mixing and inactivating the hydroxocobalamin with sodium thiosulfate.

Implications for the Pharmacist Cyanide toxicity should be considered in patients with sudden cardiovascular collapse, especially in the appropriate context of occupational exposure (e.g., laboratory or industrial work) or in a fire victim with hemodynamic instability, elevated lactic acid, or coma. Cyanide antidotes should be immediately available in the ED.

Digoxin-specific Antibody Fragments

Digoxin-specific antibody fragments (Fab) are lifesaving agents in the management of toxicity associated with the use of digoxin and other cardioactive steroids, including digitoxin and those derived from oleander, fox glove, lily of the valley, and toad venom.[63] They are safe and effective in both adults and children with acute or chronic toxicity.[64–69]

Digitalis, the most widely used cardioactive steroid, has a narrow therapeutic index.[64,68] Cardioactive steroids act on the

heart to enhance contractility, act on the conduction system of the heart to produce a variety of effects, and also act on the autonomic nervous system. The agents' toxicity is related to an exaggeration of those effects and often involves an increase in intracellular calcium. Electrocardiographic changes secondary to digoxin toxicity can be marked and highly variable.

In patients with acute digoxin poisoning, empiric treatment with digoxin-specific Fab should be considered in any patient exhibiting consequential rhythm or conduction disturbances, including symptomatic bradycardia or progressive heart block unresponsive to atropine; ventricular arrhythmias such as ventricular tachycardia or fibrillation; or a serum potassium concentration of >5.0 meq/L in the absence of another identifiable cause.[70,71] Significant nausea and vomiting after an acute digoxin overdose might also warrant the use of digoxin-specific Fab, since conduction disturbances are likely to follow. This form of therapy also should be considered if there is firm evidence of ingestion of >4 mg of digoxin by a child or >10 mg by an adult, as those total body loads of digoxin will almost certainly cause significant cardiac toxicity as the digoxin moves from the blood compartment to the heart.

The indications for digoxin-specific Fab therapy in cases of chronic digoxin poisoning are less clear but similar to those for cases of acute poisoning. Treatment with digoxin-specific Fab should be considered in any patient with a life-threatening or potentially life-threatening dysrhythmia, including severe sinus bradycardia or heart block unresponsive to atropine, as well as ventricular ectopy, tachycardia, or fibrillation.[71–73] GI complaints are less common in the context of chronic digoxin poisoning, but confusion and an altered mental status are more frequent in the elderly and might suggest the need for digoxin-specific Fab in a patient with a chronically elevated serum digoxin concentration (>2.5 ng/mL). Patients at risk for chronic digoxin toxicity include elderly patients with declining renal function, patients who have received inappropriate dosages of digoxin, patients with electrolyte abnormalities, and patients administered drugs known to inhibit the elimination of digoxin.

Digoxin-specific Fab is generally well tolerated. The adverse-effect profile includes the potential for hypokalemia, worsening of heart failure, a rapidly conducted ventricular rate, and, rarely, allergic reactions.[64]

The dosage calculation for digoxin-specific Fab can be made according to the known ingested digoxin dose, according to the serum digoxin concentration, or empirically. The empiric dosing for acute toxicity is 10–20 vials (each 38- or 40-mg vial binds 0.5 mg of digoxin); the empiric dosing for chronic toxicity is 3–5 vials for adults and 1–2 vials for children.[73] To calculate a dosage using a known serum digoxin concentration, the concentration (in nanograms per milliliter) is multiplied by the patient's weight (in kilograms) and divided by 100; the result is rounded up to the nearest integer to arrive at the required number of vials.

Implications for the Pharmacist The goal of treatment with digoxin-specific Fab is to reverse digoxin-induced

cardiotoxicity. Monitoring should include electrocardiography and serum potassium determinations. Once digoxin-specific

Fab has been administered, serum digoxin concentrations are no longer useful in dosage calculation, as there is a

resultant increase in the total digoxin concentration; therefore, repeat digoxin concentrations should not be obtained for 24 hours.[74,75]

Flumazenil

The intentional ingestion of benzodiazepines is a common cause of overdoses.[2] Flumazenil is a competitive antagonist at the benzodiazepine-receptor binding site on γ-aminobutyric acid-A. Typically, when benzodiazepines are ingested in overdose, the patient exhibits a toxidrome, or toxic syndrome, of CNS depression with relatively normal vital signs. Deaths attributed solely to the oral ingestion of benzodiazepines are rare.

While the idea of using flumazenil to reverse benzodiazepine toxicity may be tempting, the risks usually outweigh the benefits.[76,77] In a benzodiazepine-dependent patient, flumazenil can precipitate symptoms of benzodiazepine withdrawal, including seizures.[77–80] Additionally, in cases of multiple-drug ingestion, flumazenil may remove the protective effect of the benzodiazepine and unmask cardiac arrhythmias and seizures.[76,77,80] Therefore, the use of flumazenil in overdose patients is discouraged unless it can be determined with certainty that only a benzodiazepine was

ingested and that the patient is not benzodiazepine dependent and has no history of seizure.

Flumazenil also may serve a role in the treatment of children who present with altered mental status in whom possible ingestion of a benzodiazepine is suspected as the sole toxic exposure. In this scenario, invasive diagnostic techniques such as computed tomography of the head and lumbar puncture may be avoided. In such cases, flumazenil therapy will not reduce the required ED observation time; but if the child improves clinically, flumazenil can help confirm the diagnosis of benzodiazepine toxicity.

Flumazenil can also be used to reverse CNS depression associated with benzodiazepine administration during procedural sedation if the patient is known not to be benzodiazepine dependent. The initial dosage of flumazenil is 0.2 mg/min administered via a slow i.v. infusion. In the context of conscious sedation, many patients respond to total doses of 0.4 mg while some patients may require a total dose of up to 1 mg.[81] The reversal of benzodiazepine toxicity occurs rapidly after flumazenil administration; if resedation occurs, doses can be repeated at intervals of no less than 20 minutes. Resedation after flumazenil therapy is most likely to develop if >10 mg of midazolam or a longer-acting benzodiazepine is used for conscious sedation. No more than 3 mg of flumazenil should be given in one hour. In general, if resedation is not observed within two hours of the administration of a 1-mg dose of flumazenil, subsequent serious resedation is unlikely.[81,82]

Implications for the Pharmacist Due to the associated risk–benefit ratio, flumazenil is rarely indicated in the management of acutely poisoned patients.[76] These patients often have an unclear history, which makes the administration of flumazenil potentially dangerous. Flumazenil does not consistently reverse hypoventilation secondary to benzodiazepine use.[80] In the rare instances when flumazenil may be considered, it is important to ascertain that the patient is not taking benzodiazepines chronically, has a normal electrocardiogram, and is not experiencing toxicity due to a polydrug ingestion. In the context of reversal of conscious sedation, it is important to ensure that the patient has no contraindications to flumazenil, as described above.

Intravenous Fat Emulsion

Intravenous fat emulsion (IFE) has long been used to supply calories in the form of free fatty acids to patients requiring parenteral nutrition. More recently, fatty acid emulsion has been used as an antidote for drug-induced cardiovascular collapse;[83–88] the first supportive studies were laboratory investigations demonstrating the successful use of fatty acid emulsion in increasing the lethal threshold in animal models of bupivacaine-induced cardiac toxicity.[85,88,89] Since those early animal studies, there have been multiple case reports of patients successfully resuscitated after cardiovascular collapse due to toxicity from local anesthetics.[80–84] Those promising results led investigators to hypothesize that IFE would produce similar results in other scenarios of drug toxicity caused by lipid-soluble drugs such as CCBs, β-blockers, and tricyclic antidepressants.[95–98] A case report by Sirianni et al.[99] demonstrated the role of IFE in reversing the effects of an intentional overdose of bupropion and lamotrigine. The mechanism of action has not been precisely elucidated, but the "lipid sink" theory (i.e., lipophilic molecules of a local anesthetic partition into a lipemic plasma compartment, making them unavailable to the tissue) is foremost at this time;[95] other actions that might contribute to the beneficial effects of IFE include the direct activation of myocardial calcium channels[100] and the modulation of myocardial energy by providing the heart with energy in the form of fatty acids.[84,95]

Despite the promising case reports, research on the risks and benefits of IFE as an antidote for toxin-induced cardiovascular collapse remains in the discovery phase. Recently reported data from animal studies suggest that IFE has very limited adverse effects at the doses currently recommended.[101] Potential IFE-related adverse events include the development of fat embolism, or sludging, as well as interference with certain laboratory analyses due to the resultant lipemic blood;[86] other unknowns include the potential for drug interactions with other therapeutic interventions such as HIET.

IFE should be considered a first-line antidote for bupivacaine- induced toxicity.[86,87,90–93] It should also be considered in a patient with presumed toxin-induced cardiovascular collapse after the failure of advanced supportive care measures, including other accepted antidotal therapy. In addition to local anesthetics, potentially toxic agents that should be considered possibly amenable to IFE therapy include those that are lipophilic and toxic to the myocardium (e.g., tricyclic

antidepressants; CCBs, especially verapamil and diltiazem; bupropion; propranolol).

The use of IFE for the reversal of cardiotoxicity is not approved by FDA, and the dosing of IFE for this indication is unclear. Based on the published case reports of the successful use of IFE, a reasonable dosing strategy in adults is 20% IFE 100 mL (or 1.5 mL/kg) administered over 1–2 minutes by i.v. push. The bolus dose can be repeated if necessary.[99] Some reports described the use of a continuous infusion of IFE at a rate of 0.25–0.5 mL/kg/min for 30–60 minutes after the bolus dose. The maximum dose of IFE has not been established.[83,84]

Implications for the Pharmacist IFE has changed the management of bupivacaine-induced cardiotoxicity.[87] Before the use of IFE, patients with cardiac arrest secondary to bupivacaine use were rarely resuscitated.[92,93] Today, IFE should be used in any patient with bupivacane-induced cardiac toxicity. The likely scenario for IFE use in the ED is treatment of a patient experiencing toxin-related cardiovascular collapse who is not improving with aggressive standard resuscitation measures and other accepted antidotal therapies. Toxins that are lipophilic and cause cardiac toxicity are most likely to respond to IFE therapy, but data supporting such use of IFE are limited.

N-acetylcysteine

N-acetylcysteine (NAC) is a lifesaving therapy in the management of acetaminophen poisoning.[102–109] While acetaminophen is still present in the plasma, NAC acts as an antidote, primarily by replenishing glutathione stores. Secondarily, it acts as a glutathione substitute and replenishes sulfate. These mechanisms of action all serve to either limit the formation of the toxic metabolite or to detoxify it; in this way, if NAC is administered in a timely fashion, acetaminophen toxicity can be prevented.[108,109]

The Rumack-Matthew[106] nomogram is used to predict whether patients will develop hepatotoxicity, defined as a serum AST concentration of >1000 units/L, based on an initial plasma acetaminophen concentration obtained four or more hours after a single acute ingestion of acetaminophen. Indications for the initiation of NAC include a serum acetaminophen level on or above the Rumack-Matthew nomogram; situations in which a serum acetaminophen level is not available within eight hours of a potentially toxic ingestion; and hepatotoxicity, as defined by clinical symptoms or liver enzyme elevations above baseline.

Once fulminant hepatic failure has occurred, whether it is acetaminophen related or not, and even when all of the acetaminophen has already been metabolized, i.v. NAC therapy is still beneficial and may be life saving. In patients with acetaminophen-induced fulminant hepatic failure, i.v. NAC decreased mortality by 50% relative to use of a placebo.[110] NAC is believed to work through antioxidant and antiinflammatory effects, some related to glutathione formation, to improve oxygen delivery and utilization. Intravenous NAC improves cerebral, cardiac, and renal blood flow, resulting in the improved function of extrahepatic organs. NAC may also be beneficial in preventing or treating the hepatotoxicity associated with carbon tetrachloride,[111,112] Amanita phalloides,[113] and other toxins.[114,115]

Although no head-to-head studies comparing i.v. and oral NAC have been published, both routes are believed to be equally efficacious when NAC is administered within eight hours of an acetaminophen overdose;[116] NAC is effective and beneficial when given later, although the rate of hepatotoxicity increases. However, only i.v. NAC is demonstrated to be beneficial in patients with fulminant hepatic failure.[110,117] Theoretically, oral administration should provide a higher concentration of NAC to the liver due to the high extraction ratio, and i.v. dosing should provide a higher serum NAC concentration that may be more beneficial at extrahepatic sites.

The FDA-approved dosing of i.v. NAC is 150 mg/kg as a loading dose infused over 1 hour followed by 50 mg/kg over 4 hours and 100 mg/kg over 16 hours. Anaphylactoid reactions can occur, particularly with the loading dose, which is a concern in the ED; the manufacturer recommends that the loading dose be given over 60 minutes to minimize this risk.[118] In the event of an anaphylactoid reaction, discontinuing the infusion is the first step, to be followed by supportive therapy. Once the reaction abates, i.v. NAC therapy can be restarted at a much slower infusion rate or oral NAC can be administered. In patients with severe reactive airway disease, oral NAC (which rarely produces an anaphylactoid reaction) might be preferred to i.v. NAC. Improper dilution or dosing has resulted in overdoses of NAC, leading to hyponatremia,

cerebral edema, and death.[105,110,117,119]

The FDA-approved dosing of oral NAC is 140 mg/kg as a loading dose followed by 70 mg/kg every four hours for a total of 17 doses. The oral dose must be repeated if emesis occurs within one hour. Although oral NAC is rarely associated with anaphylactoid reactions, nausea and vomiting occur frequently, may delay the time to administration of an effective dose, and often require the administration of an antiemetic.[120,121]

The benefits of NAC outweigh the risks in pregnant patients with acetaminophen toxicity who meet the criteria for NAC administration. Although there are conflicting data, i.v. NAC is often recommended with the belief that i.v. NAC more readily crosses the placenta.[122,123]

NAC should not be discontinued until the acetaminophen concentration is undetectable or lower than the level of sensitivity; the AST concentration is normal or significantly improved; the synthetic function of the liver has improved, as evidenced by an International Normalized Ratio of <2; and there is no evidence of an altered mental status due to hepatic encephalopathy.[124]

Implications for the Pharmacist The decision to treat with NAC must be made quickly and based on multiple factors, including the determination of a toxic acetaminophen level and the time since overdose. NAC is nearly 100% hepatoprotective if given within the first eight hours of an overdose, and its efficacy decreases every hour after that.[108] Although most effective if given early, NAC has a clear role in late therapy and has been shown to decrease mortality.[117,125]

NAC therapy should be continued until acetaminophen is undetectable, the AST concentration is improving, and evidence of hepatic failure is no longer present. If treatment is necessary beyond the standard 21-hour dosing regimen for i.v. NAC, the third part of the protocol (6.25 mg/kg/hr) should be continued. Unusually high or prolonged serum acetaminophen concentrations may require a change in the NAC protocol; consultation with a poison center is critical.

Naloxone

The most commonly used antidote in the ED setting,[2] naloxone is a competitive antagonist at all opioid receptors, including the μ-opioid receptor.[126,127] It is a well-established antidote for respiratory depression secondary to opioid toxicity. The efficacy and safety of naloxone are dose dependent. In a previously opioid-naive patient experiencing an opioid overdose, even high doses of naloxone can be given safely.[128] However, in an opioid-dependent patient, an inappropriate dose of naloxone has the potential to precipitate opioid withdrawal and to produce or worsen acute lung injury. Therefore, it is advisable to start with a dose of 0.04 or 0.05 mg in all patients and adjust the dose upward in increments of 0.04–0.05 mg; that approach can safely reverse the respiratory depressant effects of the opioids without producing unwanted and potentially dangerous withdrawal. Opioid withdrawal with abrupt catecholamine release, especially in an apneic patient, is likely to induce vomiting and aspiration or acute lung injury.

Although case reports have suggested that naloxone may be useful in reversing toxicity due to clonidine,[129,130] ibuprofen,[131] valproic acid,[132,133] and captopril,[134] the reported effects were quite variable, often minor, and usually inadequate.

Buprenorphine is a partial μ-agonist whose toxic effects cannot be easily reversed with naloxone. Buprenorphine has a very high affinity for the μ-receptor and most likely affects μ-receptor subtypes in a dose-dependent and variable manner; this makes reversal with naloxone tricky, and a bell-shaped dose–response curve has been described, indicating that a dose too low or too high will be ineffective. In one experimental model, adults required an i.v. naloxone loading dose of 2– 3 mg followed by a continuous infusion of 4 mg/hr for one hour before the respiratory depressant effects of buprenorphine were reversed;[135–139] dosages greater than 4 mg/hr were actually ineffective, consistent with a bell-shaped dose– response relationship.

The i.v. route of naloxone administration is preferred due to a predictable, quick, and titratable onset of action. Oral or

sublingual administration result in poor absorption and limited effects.[140] Although naloxone is well absorbed with other parenteral routes of administration (intralingual, endotracheal, intranasal, subcutaneous, and intramuscular), a delayed onset of action or difficulty in titrating the dose makes those routes less desirable.

An initial naloxone dose of 0.04–0.05 mg i.v. in both opioid-dependent and opioid-naive adult patients is recommended.[140] Increasing the dose until reversal of respiratory depression maximizes the benefits while minimizing the potential for significant opioid withdrawal. Titration can be accomplished by doubling the dose every one to two minutes or escalating the dose from 0.05 mg to 0.1 mg to 0.4 mg to 2 mg to 10 mg. During dose titration, bag-valve mask ventilation should be used as necessary. Although there is not a consensus on the initial dose, starting with a lower dose of naloxone (0.04–0.05 mg) rather than the "standard" 2-mg dose is considered best practice. A lack of sufficient patient response to treatment with 10 mg of naloxone should call into question a diagnosis of isolated opioid toxicity. Patients experiencing an overdose of synthetic opioids (e.g., buprenorphine, fentanyl, methadone) often require higher doses of naloxone but generally respond to doses of 10 mg.

Due to the relatively short half-life of naloxone, the duration of its clinical effect is generally 30–90 minutes. This relatively brief duration of effect is critical to clinicians because the duration of effect of the opioid is frequently much longer than that of naloxone and, therefore, respiratory depression may recur. Repeat doses or a continuous infusion at an hourly dose equal to two thirds of the initial dose of naloxone may be necessary, so close monitoring of the patient is required in the ED. It is recommended that the naloxone infusion be started at two thirds of the hourly dose that was effective in reversing the respiratory depression. This is based on a pharmacokinetic study done in the 1980s.[140]

Octreotide

A long-acting synthetic analog of somatostatin, octreotide is used in toxicology to counteract the insulin-releasing properties of the sulfonylurea and miglitinide oral hypoglycemics.[141] –148 Sulfonylureas and miglitinides stimulate insulin release by binding to adenosine triphosphate (ATP)-sensitive potassium channels on the beta-islet cell of the pancreas, increasing intracellular ATP concentrations, which increases intracellular calcium concentrations. Octreotide inhibits pancreatic beta-islet cell insulin release via G-protein-coupled receptors[141,142] that inhibit cAMP production; this decreases intracellular calcium influx, thereby causing a decrease in insulin release.[143]

Octreotide is used as an adjunct to dextrose to manage hypoglycemia induced by sulfonylureas and other exogenous causes of insulin release.[142,144] –146 Octreotide limits the production of hypoglycemia and decreases the need for supplemental dextrose in such cases.

There are no published data from randomized clinical trials confirming the unique effects of octreotide in patients with sulfonylurea-induced hypoglycemia. One study of human volunteers confirmed the ability of octreotide to reduce dextrose requirements in fasting patients given sulfonylureas.[141] Case reports described the reversal of sulfonylurea-induced hypoglycemia with octreo-tide in both adults and children.[147] Vallurupalli[148] described two patients with sulfonylurea- induced hypoglycemia and congestive heart failure who had blood glucose concentrations of 31 and 36 mg/dL, respectively, due to inadequate oral intake of glucose and underlying renal failure. Despite repeat doses of dextrose, both patients remained hypoglycemic. One of those patients received two doses of octreotide 50 μg 12 hours apart; after the initial dose, the first blood glucose value was 62 mg/dL, and the concentration rose to 121 mg/dL after the second dose. In the second case reported by Vallurupalli, the patient initially received 25 μg of octreotide subcutaneously, but the hypoglycemia persisted until two additional doses of 50 μg given 12 hours apart were administered.

Adverse effects associated with several doses of octreotide are minimal and may include diarrhea and abdominal discomfort. Octreotide has a relatively benign adverse-effect profile with short-term use, and it is therefore recommended that octreotide be considered in any patient with recurrent hypoglycemia after a single dose of i.v. hypertonic dextrose (0.5–1 g/kg) when the differential diagnosis includes sulfonylurea toxicity. The use of i.v. dextrose should be followed by feeding the patient.

Subcutaneous administration is the most frequently described method of octreotide delivery in the scientific


literature.[142,146] The usual adult dose of octreotide is 50 μg subcutaneously, with doses repeated every six hours as needed.

Implications for the Pharmacist Octreotide decreases insulin release from the pancreas when secondary to an insulin secretatogue. In otherwise healthy patients (i.e., patients without diabetes) with an intact pancreas, the administration of exogenous dextrose to reverse the hypoglycemia caused by ingestion of an oral hypoglycemic induces the pancreas to release more insulin, which can exacerbate the hypoglycemia. Some clinicians advocate the use of octreotide after the first hypoglycemic episode, although most suggest that it should be initiated after the second hypoglycemic event.[142] The duration of effect of the ingested xenobiotic causing the hypoglycemia should help determine the initial number of doses of octreotide and the duration of monitoring after the last dose of octreotide.

Conclusion

Pharmacists can play a key role in reducing poisoning and overdose injuries and deaths by assisting in the early recognition of toxic exposures and guiding emergency personnel on the proper storage, selection, and use of antidotal therapies.

References

  1. Centers for Disease Control and Prevention. Unintentional poisoning deaths— United States, 1999–2004. MMWR Morb Mortal Wkly Rep. 2007; 56:93–6.

  2. Bronstein AC, Spyker DA, Cantilena LR Jr et al. 2009 annual report of the American Association of Poison Control Centers' National Poison Data System (NPDS): 27th annual report. Clin Toxicol (Phila). 2010; 48:979–1178.

  3. Linakis JG, Frederick KA. Poisoning deaths not reported to the regional poison center. Ann Emerg Med. 1993; 22:1822–8.

  4. Shepherd G, Klein-Schwartz W. Accidental and suicidal adolescent poisoning deaths in the US, 1979–1994. Arch Pediatr Adolesc Med. 1998; 152:1181–5.

  5. Dart RC, Borron SW, Caravati EM et al. Antidote Summit Authorship G. Expert consensus guidelines for stocking of antidotes in hospitals that provide emergency care. Ann Emerg Med. 2009; 54:386–94.e1.

  6. Brent J, McMartin K, Phillips S et al., for the Methylpyrazole for Toxic Alcohols Study Group. Fomepizole for the treatment of methanol poisoning. N Engl J Med. 2001; 344:424–9.

  7. Brent J, McMartin K, Phillips S et al., for the Methylpyrazole for Toxic Alcohols Study Group. Fomepizole for the treatment of ethylene glycol poisoning. N Engl J Med. 1999; 340:832–8.

  8. Barceloux DG, Bond GR, Krenzelok EP et al., for the American Academy of Clinical Toxicology Ad Hoc Committee on the Treatment Guidelines for Methanol Poisoning. American Academy of Clinical Toxicology practice guidelines on the treatment of methanol poisoning. J Toxicol Clin Toxicol. 2002; 40:415–46.

  9. Bronstein AC, Spyker DA, Cantilena LR Jr et al. 2008 annual report of the American Association of Poison Control Centers' National Poison Data System (NPDS): 26th annual report. Clin Toxicol (Phila). 2009; 47:911–1084.

  10. McMartin KE, Brent J, META Study Group. Pharmacokinetics of fomepizole (4MP) in patients. J Toxicol Clin

    Toxicol. 1998; 36:450–1.

  11. Sanaei-Zadeh H, Zamani N, Shadnia S. Outcomes of visual disturbances after methanol poisoning. Clin Toxicol.

    2011; 49:102–7.

  12. McMartin KE, Heath A. Treatment of ethylene glycol poisoning with intravenous 4-methylpyrazole. N Engl J Med.

    1989; 320:125.

  13. Hanif M, Mobarak MR, Ronan A et al. Fatal renal failure caused by diethylene glycol in paracetamol elixir: the

    Bangladesh epidemic. BMJ. 1995; 311:88–91.

  14. Okuonghae HO, Ighogboja IS, Lawson JO et al. Diethylene glycol poisoning in Nigerian children. Ann Trop

    Paediatr. 1992; 12:235–8.

  15. Rollins YD, Filley CM, McNutt JT et al. Fulminant ascending paralysis as a delayed sequela of diethylene glycol

    (Sterno) ingestion. Neurology. 2002; 59:1460–3.

  16. Marraffa JM, Holland MG, Stork CM et al. Diethylene glycol: widely used solvent presents serious poisoning

potential. J Emerg Med. 2007; 35:401–6.

  1. Hasbani MJ, Sansing LH, Perrone J et al. Encephalopathy and peripheral neuropathy following diethylene glycol

    ingestion. Neurology. 2005; 64:1273–5.

  2. Alfred S, Coleman P, Harris D et al. Delayed neurologic sequelae resulting from epidemic diethylene glycol

    poisoning. Clin Toxicol (Phila). 2005; 43:155–9.

  3. Hari P, Jain Y, Kabra SK. Fatal encephalopathy and renal failure caused by diethylene glycol poisoning. J Trop

    Pediatr. 2006; 52:442–4.

  4. Landry GM, Martin S, McMartin KE. Diglycolic acid is the nephrotoxic metabolite in diethylene glycol poisoning

    inducing necrosis in human proximal tubule cells in vitro. Toxicol Sci. 2011; 123:374–83.

  5. Jacobsen D, McMartin KE. Antidotes for methanol and ethylene glycol poisoning. J Toxicol Clin Toxicol. 1997;

    35:127–43.

  6. Wallemacq PE, Vanbinst R, Haufroid V et al. Plasma and tissue determination of 4-methylpyrazole for

    pharmacokinetic analysis in acute adult and pediatric methanol/ethylene glycol poisoning. Ther Drug Monit. 2004;

    26:258–62.

  7. Antizol (fomepizole) injection product information. Palo Alto, CA: Jazz Pharmaceuticals, Inc.; 2007 Jan.

  8. Noker PE, Eells JT, Tephly TR. Methanol toxicity: treatment with folic acid and 5-formyl tetrahydrofolic acid. Alcohol

    Clin Exp Res. 1980; 4:378–83.

  9. Noker PE, Tephly TR. The role of folates in methanol toxicity. Adv Exp Med Biol. 1980; 132:305–15.

  10. Nath R, Thind SK, Murthy MS et al. Role of pyridoxine in oxalate metabolism. Ann N Y Acad Sci. 1990; 585:274–

    84.

  11. Sharma S, Sidhu H, Narula R et al. Comparative studies on the effect of vitamin A, B1 and B6 deficiency on oxalate

    metabolism in male rats. Ann Nutr Metab. 1990; 34:104–11.

  12. Kerns W II. Management of betaadrenergic blocker and calcium channel antagonist toxicity. Emerg Med Clin North

    Am. 2007; 25:309–31.

  13. DeWitt CR, Waksman JC. Pharmacology, pathophysiology and management of calcium channel blocker and beta-

    blocker toxicity. Toxicol Rev. 2004; 23:223–38.

  14. Levine M, Boyer EW, Pozner CN et al. Assessment of hyperglycemia after calcium channel blocker overdoses

    involving diltiazem or verapamil. Crit Care Med. 2007; 35:2071–5.

  15. Boyer EW, Duic PA, Evans A. Hyperinsulinemia/euglycemia therapy for calcium channel blocker poisoning. Pediatr

    Emerg Care. 2002; 18:36–7.

  16. Yuan TH, Kerns WP II, Tomaszewski CA et al. Insulin-glucose as adjunctive therapy for severe calcium channel

    antagonist poisoning. J Toxicol Clin Toxicol. 1999; 37:463–74.

  17. Farah AE, Alousi AA. The actions of insulin on cardiac contractility. Life Sci. 1981; 29:975–1000.

  18. Mégarbane B, Karyo S, Baud FJ. The role of insulin and glucose (hyperinsulinaemia/euglycaemia) therapy in acute

    calcium channel antagonist and beta-blocker poisoning. Toxicol Rev. 2004; 23:215–22.

  19. Lheureux PE, Zahir S, Gris M et al. Benchto- bedside review: hyperinsulinaemia/euglycaemia therapy in the

    management of overdose of calcium-channel blockers. Crit Care. 2006; 10:212.

  20. Kline JA, Leonova E, Raymond RM. Beneficial myocardial metabolic effects of insulin during verapamil toxicity in

    the anesthetized canine. Crit Care Med. 1995; 23:1251–63.

  21. Kline JA, Tomaszewski CA, Schroeder JD et al. Insulin is a superior antidote for cardiovascular toxicity induced by

    verapamil in the anesthetized canine. J Pharmacol Exp Ther. 1993; 267:744–50.

  22. Kerns W II, Ransom M, Tomaszewski C et al. The effects of extracellular ions on beta-blocker cardiotoxicity.

    Toxicol Appl Pharmacol. 1996; 137:1–7.

  23. Kerns W. Antidotes in depth: insulineuglycemia therapy. In: Nelson LS, Lewin NA, Howland MA et al., eds.

    Goldfrank's toxicologic emergencies. 9th ed. New York: McGraw-Hill; 2011:893–5.

  24. Engebretsen KM, Kac Z, Marek KK et al. High dose insulin therapy in betablocker and calcium channel blocker

    poisoning. Clin Toxicol. 2011; 49:277–83.

  25. Cyanide antidote kit (sodium nitrite injection, sodium theosulfate injection, amyl nitrite inhalants) package insert.

    Lake Forest, IL: Akorn, Incorporated; 2008.

  26. Nithiodote (sodium nitrite injection and sodium thiosulfate injection) package insert. Scottsdale, AZ: Hope

Pharmaceuticals; 2011.

  1. Cyanokit (hydroxocobalamin for injection) product information. Semoy, France: Merck Sante; 2009.

  2. Barillo DJ, Goode R, Esch V. Cyanide poisoning in victims of fire: analysis of 364 cases and review of the literature.

    J Burn Care Rehabil. 1994; 15:46–57.

  3. Eckstein M, Maniscalco PM. Focus on smoke inhalation—the most common cause of acute cyanide poisoning.

    Prehospital Disaster Med. 2006; 21:S49–55.

  4. Fortin JL, Waroux S, Giocanti JP et al. Hydroxocobalamin for poisoning caused by ingestion of potassium cyanide:

    a case study. J Emerg Med. 2010; 39:320–4.

  5. Baud FJ, Borron SW, Mégarbane B et al. Value of lactic acidosis in the assessment of the severity of acute cyanide

    poisoning. Crit Care Med. 2002; 30:2044–50.

  6. Chen KK, Rose C, Clowes G. Amyl nitrite and cyanide poisoning. JAMA. 1933; 100:1921–2.

  7. Chen KK, Rose C. Nitrite and thiosulfate therapy in cyanide poisoning. JAMA. 1952; 149:113–9.

  8. Hall AH, Kulig KW, Rumack BH. Suspected cyanide poisoning in smoke inhalation and complications of sodium

    nitrite therapy. J Toxicol Clin Exp. 1989; 9:3–9.

  9. Klimmek R, Krettek C, Werner HW. Ferrihaemoglobin formation by amyl nitrite and sodium nitrite indifferent species

    in vivo and in vitro. Arch Toxicol. 1988; 62:152–60.

  10. Curry SC, Carlton MW, Raschke RA. Prevention of fetal and maternal cyanide toxicity from nitroprusside with

    coinfusion of sodium thiosulfate in gravid ewes. Anesth Analg. 1997; 84:1121–6.

  11. Borron SW, Baud FJ, Mégarbane B et al. Hydroxocobalamin for severe acute cyanide poisoning by ingestion or

    inhalation. Am J Emerg Med. 2007; 25:551–8.

  12. Fortin JL, Giocanti JP, Ruttimann M et al. Prehospital administration of hydroxocobalamin for smoke

    inhalationassociated cyanide poisoning: 8 years of experience in the Paris fire brigade. Clin Toxicol (Phila). 2006;

    44(suppl 1):37–44.

  13. Borron SW, Stonerook M, Reid F. Efficacy of hydroxocobalamin for the treatment of acute cyanide poisoning in

    adult beagle dogs. Clin Toxicol (Phila). 2006; 44(suppl 1):5–15.

  14. Uhl W, Nolting A, Golor G et al. Safety of hydroxocobalamin in healthy volunteers in a randomized, placebo-

    controlled study. Clin Toxicol (Phila). 2006; 44(suppl 1):17–28.

  15. Sutter M, Tereshchenko N, Rafii R et al. Hemodialysis complications of hydroxocobalamin: a case report. J Med

    Toxicol. 2010; 6:165–7.

  16. Cescon DW, Juurlink DN. Discoloration of skin and urine after treatment with hydroxocobalamin for cyanide

    poisoning. CMAJ. 2009; 180:251.

  17. Curry SC, Connor DA, Raschke RA. Effect of the cyanide antidote hydroxocobalamin on commonly ordered serum

    chemistry studies. Ann Emerg Med. 1994; 24:65–7.

  18. Beaudeux JL, Flourie F, Peynet J et al. Hydroxocobalamin and sodium thiosulfate interfere negatively with

    measurement of creatine kinase activity. Clin Chem. 1994; 40:2120–1.

  19. Beckerman N, Leikin SM, Aitchinson R et al. Laboratory interferences with the newer cyanide antidote:

    hydroxocobalamin. Semin Diagn Pathol. 2009; 26:49–52.

  20. Pamidi PV, DeAbreu M, Kim D et al. Hydroxocobalamin and cyanocobalamin interference on co-oximetry based

    hemoglobin measurements. Clin Chim Acta. 2009; 401:63–7.

  21. Centers for Disease Control and Prevention. Deaths associated with a purported aphrodisiac—New York City,

    February 1993–May 1995. MMWR Morb Mortal Wkly Rep. 1995; 44:853–5.

  22. Hickey AR, Wenger TL, Carpenter VP et al. Digoxin immune fab therapy in the management of digitalis

    intoxication: safety and efficacy results of an observational surveillance study. J Am Coll Cardiol. 1991; 17:590–8.

  23. Woolf AD, Wenger TL, Smith TW et al. Results of multicenter studies of digoxin-specific antibody fragments in

    managing digitalis intoxication in the pediatric population. Am J Emerg Med. 1991; 9(2, suppl 1):16,20 [discussion

    33- 4].

  24. Antman EM, Wenger TL, Butler VP Jr et al. Treatment of 150 cases of lifethreatening digitalis intoxication with

    digoxin-specific fab antibody fragments. Final report of a multicenter study. Circulation. 1990; 81:1744–52.

  25. Lapostolle F, Borron SW, Verdier C et al. Assessment of digoxin antibody use in patients with elevated serum

    digoxin following chronic or acute exposure. Intensive Care Med. 2008; 34:1448–53.

  26. Lapostolle F, Borron SW, Verdier C et al. Digoxin-specific fab fragments as single first-line therapy in digitalis

poisoning. Crit Care Med. 2008; 36:3014–8.

  1. Slifman NR, Obermeyer WR, Aloi BK et al. Contamination of botanical dietary supplements by digitalis lanata. N Engl J Med. 1998; 339:806–11.

  2. Bismuth C, Borron SW, Baud FJ et al. Immunotoxicotherapy: successes, disappointments and hopes. Hum Exp Toxicol. 1997; 16:602–8.

  3. Domart Y, Bismuth C, Schermann JM et al. Digitoxin poisoning: reversing ventricular fibrillation with fab fragments of anti-digoxin antibody. Nouv Presse Med. 1982; 11:3827–30.

  4. Smolarz A, Roesch E, Lenz E et al. Digoxin specific antibody (fab) fragments in 34 cases of severe digitalis intoxication. J Toxicol Clin Toxicol. 1985; 23:327–40.

  5. Borron SW, Bismuth C, Muszynski J. Advances in the management of digoxin toxicity in the older patient. Drugs Aging. 1997; 10:18–33.

  6. Doherty JE, Perkins WH, Flanigan WJ. The distribution and concentration of tritiated digoxin in human tissues. Ann Intern Med. 1967; 66:116–24.

  7. Gibb T, Adams PC, Parnham AJ et al. Plasma digoxin: assay anomalies in Fabtreated patients. Br J Clin Pharmacol. 1983; 16:445–7.

  8. Goldfrank LR. Flumazenil: a pharmacologic antidote with limited medical toxicology utility, or ... an antidote in search of an overdose. Acad Emerg Med. 1997; 4:935–6.

  9. Haverkos GP, DiSalvo RP, Imhoff TE. Fatal seizures after flumazenil administration in a patient with mixed overdose. Ann Pharmacother. 1994; 28:1347–9.

  10. Lheureux P, Vranckx M, Leduc D et al. Risks of flumazenil in mixed benzodiazepine-tricyclic antidepressant overdose: report of a preliminary study in the dog. J Toxicol Clin Exp. 1992; 12:43–53.

  11. Mordel A, Winkler E, Almog S et al. Seizures after flumazenil administration in a case of combined benzodiazepine and tricyclic antidepressant overdose. Crit Care Med. 1992; 20:1733–4.

  12. Seger DL. Flumazenil—treatment or toxin. J Toxicol Clin Toxicol. 2004; 42:209–16.

  13. McEvoy GK, Snow EK, Miller J et al., eds. AHFS drug information 2009. Bethesda, MD: American Society of

    Health-System Pharmacists; 2009:2.

  14. Romazicon (flumazenil) injection product information. Nutley, NJ: Roche Pharmaceuticals, Inc.; 2007.

  15. Weinberg G. Lipid rescue resuscitation from local anaesthetic cardiac toxicity. Toxicol Rev. 2006; 25:139–45.

  16. Weinberg G. Lipid infusion resuscitation for local anesthetic toxicity: proof of clinical efficacy. Anesthesiology. 2006;

    105:7–8.

  17. Weinberg G, Ripper R, Feinstein DL et al. Lipid emulsion infusion rescues dogs from bupivacaine-induced cardiac

    toxicity. Reg Anesth Pain Med. 2003; 28:198–202.

  18. Weinberg GL. Limits to lipid in the literature and lab: what we know, what we don't know. Anesth Analg. 2009;

    108:1062–4.

  19. Weinberg GL. Lipid infusion therapy: translation to clinical practice. Anesth Analg. 2008; 106:1340–2.

  20. Weinberg GL, di Gregorio G, Ripper R et al. Resuscitation with lipid versus epinephrine in a rat model of

    bupivacaine overdose. Anesthesiology. 2008; 108:907–13

  21. Hiller DB, Gregorio GD, Ripper R et al. Epinephrine impairs lipid resuscitation from bupivacaine overdose: a

    threshold effect. Anesthesiology. 2009; 111:498–505.

  22. Charbonneau H, Marcou TA, Mazoit JX et al. Early use of lipid emulsion to treat incipient mepivacaine intoxication.

    Reg Anesth Pain Med. 2009; 34:277–8.

  23. Litz RJ, Roessel T, Heller AR et al. Reversal of central nervous system and cardiac toxicity after local anesthetic

    intoxication by lipid emulsion injection. Anesth Analg. 2008; 106:1575–7.

  24. Markowitz S, Neal JM. Immediate lipid emulsion therapy in the successful treatment of bupivacaine systemic

    toxicity. Reg Anesth Pain Med. 2009; 34:276.

  25. Rosenblatt MA, Abel M, Fischer GW et al. Successful use of a 20% lipid emulsion to resuscitate a patient after a

    presumed bupivacaine-related cardiac arrest. Anesthesiology. 2006; 105:217–8.

  26. Weinberg G, Hertz P, Newman J. Lipid, not propofol, treats bupivacaine overdose. Anesth Analg. 2004; 99:1875–6.

  27. Bania TC, Chu J, Perez E et al. Hemodynamic effects of intravenous fat emulsion in an animal model of severe

    verapamil toxicity resuscitated with atropine, calcium, and saline. Acad Emerg Med. 2007; 14:105–11.

  28. Harvey M, Cave G, Hoggett K. Correlation of plasma and peritoneal diasylate clomipramine concentration with

hemodynamic recovery after intralipid infusion in rabbits. Acad Emerg Med. 2009; 16:151–6.

  1. Perez E, Bania TC, Medlej K et al. Determining the optimal dose of intravenous fat emulsion for the treatment of severe verapamil toxicity in a rodent model. Acad Emerg Med. 2008; 15:1284–9.

  2. Tebbutt S, Harvey M, Nicholson T et al. Intralipid prolongs survival in a rat model of verapamil toxicity. Acad Emerg Med. 2006; 13:134–9.

  3. Sirianni AJ, Osterhoudt KC, Calello DP et al. Use of lipid emulsion in the resuscitation of a patient with prolonged cardiovascular collapse after overdose of bupropion and lamotrigine. Ann Emerg Med. 2008; 51:412–5.

  4. Gueret G, Pennec JP, Arvieux CC. Hemodynamic effects of intralipid after verapamil intoxication may be due to a direct effect of fatty acids on myocardial calcium channels. Acad Emerg Med. 2007; 14:761.

  5. Hiller DB, di Gregorio G, Kelly K et al. Safety of high volume lipid emulsion infusion: a first approximation of LD50 in rats. Reg Anesth Pain Med. 2010; 35:140–4.

  6. Harrison PM, Keays R, Bray GP et al. Improved outcome of paracetamolinduced fulminant hepatic failure by late administration of acetylcysteine. Lancet. 1990; 335:1572–3.

  7. Harrison PM, Wendon JA, Gimson AE et al. Improvement by acetylcysteine of hemodynamics and oxygen transport in fulminant hepatic failure. N Engl J Med. 1991; 324:1852–7.

  8. Keays R, Harrison PM, Wendon JA et al. Intravenous acetylcysteine in paracetamol induced fulminant hepatic failure: a prospective controlled trial. BMJ. 1991; 303:1026–9.

  9. Prescott LF, Illingworth RN, Critchley JA et al. Intravenous N-acetylcystine: the treatment of choice for paracetamol poisoning. Br Med J. 1979; 2:1097–100.

  10. Rumack BH, Matthew H. Acetaminophen poisoning and toxicity. Pediatrics. 1975; 55:871–6.

  11. Rumack BH, Peterson RG. Acetaminophen overdose: incidence, diagnosis, and management in 416 patients.

    Pediatrics. 1978; 62(5, pt 2, suppl):898–903.

  12. Smilkstein MJ, Knapp GL, Kulig KW et al. Efficacy of oral N-acetylcysteine in the treatment of acetaminophen

    overdose: analysis of the national multicenter study (1976 to 1985). N Engl J Med. 1988; 319:1557–62.

  13. Smilkstein MJ, Douglas DR, Daya MR. Acetaminophen poisoning and liver function. N Engl J Med. 1994;

    331:1310–1.

  14. Galicia-Moreno M, Rodríguez-Rivera A, Reyes-Gordillo K et al. N-acetylcysteine prevents carbon tetrachloride-

    induced liver cirrhosis: role of liver transforming growth factor-beta and oxidative stress. Eur J Gastroenterol

    Hepatol. 2009; 21:908–14.

  15. Valles EG, de Castro CR, Castro JA. N-acetyl cysteine is an early but also a late preventive agent against carbon

    tetrachloride-induced liver necrosis. Toxicol Lett. 1994; 71:87–95.

  16. Montanini S, Sinardi D, Praticò C et al. Use of acetylcysteine as the life-saving antidote in Amanita phalloides

    (death cap) poisoning. Case report on 11 patients. Arzneimittelforschung. 1999; 49:1044–7.

  17. Dell'Aglio DM, Sutter ME, Schwartz MD et al. Acute chloroform ingestion successfully treated with intravenously

    administered N-acetylcysteine. J Med Toxicol. 2010; 6:143–6.

  18. Anderson IB, Mullen WH, Meeker JE et al. Pennyroyal toxicity: measurement of toxic metabolite levels in two cases

    and review of the literature. Ann Intern Med. 1996; 124:726–34.

  19. Yarema MC, Johnson DW, Berlin RJ et al. Comparison of the 20-hour intravenous and 72-hour oral acetylcysteine

    protocols for the treatment of acute acetaminophen poisoning. Ann Emerg Med. 2009; 54:606–14.

  20. Acetadote (acetylcysteine) injection package insert. Nashville, TN: Cumberland Pharmaceuticals Inc.; 2011.

  21. Prescott LF, Illingworth RN, Critchley JA et al. Intravenous N-acetylcysteine: still the treatment of choice for

    paracetamol poisoning. Br Med J. 1980; 280:46–7.

  22. Heard K. A multicenter comparison of the safety of oral versus intravenous acetylcysteine for treatment of

    acetaminophen overdose. Clin Toxicol (Phila). 2010; 48:424–30.

  23. Heard KJ. Acetylcysteine for acetaminophen poisoning. N Engl J Med. 2008; 359:285–92.

  24. Horowitz RS, Dart RC, Jarvie DR et al. Placental transfer of N-acetylcysteine following human maternal

    acetaminophen toxicity. J Toxicol Clin Toxicol. 1997; 35:447–51.

  25. Selden BS, Curry SC, Clark RF et al. Transplacental transport of N-acetylcysteine in an ovine model. Ann Emerg

    Med. 1991; 20:1069–72.

  26. Dart RC, Rumack BH. Patient-tailored acetylcysteine administration. Ann Emerg Med. 2007; 50:280–1.

  27. Evans LE, Swainson CP, Roscoe P et al. Treatment of drug overdosage with naloxone, a specific narcotic

antagonist. Lancet. 1973; 1:452–5.

  1. Evans LE. Use of naloxone in opiate poisoning. Br Med J. 1973; 2:717.

  2. Dixon R, Gentile J, Hsu HB et al. Nalmefene: safety and kinetics after single and multiple oral doses of a new

    opioid antagonist. J Clin Pharmacol. 1987; 27:233–9.

  3. Jorm CM, Stamford JA. Actions of morphine on noradrenaline efflux in the rat locus coeruleus are mediated via

    both opioid and alpha 2 adrenoceptor mechanisms. Br J Anaesth. 1995; 74:73–8.

  4. Seger DL. Clonidine toxicity revisited. J Toxicol Clin Toxicol. 2002; 40:145–55.

  5. Easley RB, Altemeier WA III. Central nervous system manifestations of an ibuprofen overdose reversed by

    naloxone. Pediatr Emerg Care. 2000; 16:39–41.

  6. Roberge RJ, Francis EH III. Use of naloxone in valproic acid overdose: case report and review. J Emerg Med.

    2002; 22:67–70.

  7. Thanacoody HK. Chronic valproic acid intoxication: reversal by naloxone. Emerg Med J. 2007; 24:677–8.

  8. Varon J, Duncan SR. Naloxone reversal of hypotension due to captopril overdose. Ann Emerg Med. 1991;

    20:1125–7.

  9. Dahan A. Opioid-induced respiratory effects: new data on buprenorphine. Palliat Med. 2006; 20(suppl 1):S3–8.

  10. Dahan A, Aarts L, Smith TW. Incidence, reversal, and prevention of opioid-induced respiratory depression.

    Anesthesiology. 2010; 112:226–38.

  11. Sarton E, Teppema L, Dahan A. Naloxone reversal of opioid-induced respiratory depression with special emphasis

    on the partial agonist/antagonist buprenorphine. Adv Exp Med Biol. 2008; 605:486–91.

  12. Van Dorp E, Yassen A, Sarton E et al. Naloxone reversal of buprenorphine-induced respiratory depression.

    Anesthesiology. 2006; 105:51–7.

  13. Yassen A, Olofsen E, van Dorp E et al. Mechanism-based pharmacokinetic pharmacodynamic modelling of the

    reversal of buprenorphine-induced respiratory depression by naloxone: a study in healthy volunteers. Clin

    Pharmacokinet. 2007; 46:965–80.

  14. Goldfrank L, Weisman RS, Errick JK et al. A dosing nomogram for continuous infusion intravenous naloxone. Ann

    Emerg Med. 1986; 15:566–70.

  15. Boyle PJ, Justice K, Krentz AJ et al. Octreotide reverses hyperinsulinemia and prevents hypoglycemia induced by

    sulfonylurea overdoses. J Clin Endocrinol Metab. 1993; 76:752–6.

  16. Fasano CJ, O'Malley G, Dominici P et al. Comparison of octreotide and standard therapy versus standard therapy

    alone for the treatment of sulfonylureainduced hypoglycemia. Ann Emerg Med. 2008; 51:400–6.

  17. Calello DP, Kelly A, Osterhoudt KC. Case files of the medical toxicology fellowship training program at the

    children's hospital of Philadelphia: a pediatric exploratory sulfonylurea ingestion. J Med Toxicol. 2006; 2:19–24.

  18. McLaughlin SA, Crandall CS, McKinney PE. Octreotide: an antidote for sulfonylurea-induced hypoglycemia. Ann

    Emerg Med. 2000; 36:133–8.

  19. Green RS, Palatnick W. Effectiveness of octreotide in a case of refractory sulfonylurea-induced hypoglycemia. J

    Emerg Med. 2003; 25:283–7.

  20. Rath S, Bar-Zeev N, Anderson K et al. Octreotide in children with hypoglycaemia due to sulfonylurea ingestion. J

    Paediatr Child Health. 2008; 44:383–4.

  21. Pelavin PI, Abramson E, Pon S et al. Extended-release glipizide overdose presenting with delayed hypoglycemia

    and treated with subcutaneous octreotide. J Pediatr Endocrinol. 2009; 22:171–5.

  22. Vallurupalli S. Safety of subcutaneous octreotide in patients with sulfonylurea induced hypoglycemia and

    congestive heart failure. Ann Pharmacother. 2010; 44:387–90.

Sourced from:   American Journal of Health-System Pharmacy. 2012;69(3):199-212. © 2012 American Society of Health-System 

Review Article on Marijuana

The following is a recent review of the literature regarding marijuana.  Printed in the J. Am. Board Fam. Med., and copied in Medscape, it describes the medical uses of marijuana and its derivatives, and reviews the medical-legal aspects of this drug's usage.

Cannabis and Its Derivatives

Lawrence Leung, MBBChir, MFM(Clin)

Posted: 08/30/2011; J Am Board Fam Med. 2011;24(4):452-462. © 2011 American Board of Family Medicine

Abstract and Introduction

Abstract

Background: Use of cannabis is often an under-reported activity in our society. Despite legal restriction, cannabis is often used to relieve chronic and neuropathic pain, and it carries psychotropic and physical adverse effects with a propensity for addiction. This article aims to update the current knowledge and evidence of using cannabis and its derivatives with a view to the sociolegal context and perspectives for future research.
Methods: Cannabis use can be traced back to ancient cultures and still continues in our present society despite legal curtailment. The active ingredient, Δ9-tetrahydrocannabinol, accounts for both the physical and psychotropic effects of cannabis. Though clinical trials demonstrate benefits in alleviating chronic and neuropathic pain, there is also significant potential physical and psychotropic side-effects of cannabis. Recent laboratory data highlight synergistic interactions between cannabinoid and opioid receptors, with potential reduction of drug-seeking behavior and opiate sparing effects. Legal rulings also have changed in certain American states, which may lead to wider use of cannabis among eligible persons.
Conclusions: Family physicians need to be cognizant of such changing landscapes with a practical knowledge on the pros and cons of medical marijuana, the legal implications of its use, and possible developments in the future.

Case 1

Scenario

You are a family physician in Ontario, Canada. A 54-year-old man suffering from multiple sclerosis came to your office asking for a prescription for medical marijuana to control his pain. He was taking continuous-release morphine, gabapentin, and lamotrigine, but this combination was still insufficient. He visited Florida a few times, where he smoked cannabis, which helped tremendously to reduce the neuropathic pain and detach his mind from it. He would like to continue using cannabis but is worried about the legal implications and the purity of sample he may obtain on the street.

Suggested Management

The evidence of various forms of cannabis (smoked, oral, and oromucosal spray) for treating neuropathic pain caused by multiple sclerosis should be discussed against the known harms and challenges of usage. Sativex (legally available form of cannabis in Canada; GW Pharmaceuticals, Salisbury, Wiltshire, UK) could be recommended as a first-line treatment. If the patient still decided to pursue a smoked or oral extract of cannabis, referral should be made to recognized specialists in Quebec for a full assessment of eligibility of patient's use and possession of medical marijuana. Close monitoring of the patient would be necessary.

Case 2

Scenario

You are a family physician in the state of California. A 65-year-old male veteran came to your office as a new patient. He had a history of chronic leg pain caused by a shrapnel injury he suffered during the Vietnam War in 1968. Since the 1970s, he has been treated at the local veterans hospital under a pain management program, but control has been unsatisfactory. When asked if he used any recreational drugs, including marijuana, he evaded your question and said he needed to stay on the pain program. You suspected he was using marijuana for his chronic pain.

Suggested Management

The patient should be informed of the new directive from the Veterans Health Administration regarding veterans' use of marijuana and be reassured that he would not be denied his pain management services at the veterans hospital on that basis. He also should be encouraged to discuss his marijuana use with you so that you can monitor his progress. Liaising with an addiction medicine specialist can be helpful to ensure the best follow-up of this patient.

Cannabis, also known as marijuana, refers to the preparation 53 from the plant belonging to the family Cannabaceae, the genusCannabis, and the species Cannabis sativa, which possess psychoactive effects. The flowering tops, leaves, and stalks of the mature female plant are commonly used as the herbal form of cannabis, but sometimes the resinous extract of compressed herb is also used and is called "hash." Archaeologists have identified fibers from cannabis stems in specimens dating back to 4000 BC, and its incorporation into textiles and paper was found in the tombs of the Chinese Han dynasty (~100 BC).[1] The first record of cannabis as a medicine can be found in the oldest Chinese pharmacopeia, Shen Nong Ben Cao Jing, written in the Eastern Han Dynasty (AD 25 to AD 220), which was indicated for rheumatic pain, malaria, constipation, and disorders of the female reproductive system.[2] Though the cannabis leaf and stem is rarely used nowadays in Chinese herbal medicine, cannabis seeds, which contain very few psychoactive ingredients, are still commonly prescribed for their laxative effects.[2]Smoking cannabis is often an under-reported behavior in our society, with a reported prevalence from the World Health Organization of 3.9% among the global population aged 15 to 64 years.[3] There are more than 70 psychoactive compounds called "cannabinoids" that have been identified in cannabis,[4] among which Δ9- tetrahydrocannabinol (THC) accounts for most of the psychological and physical effects, and its content is often used as a measure of sample potency. We now know that THC acts on 2 types of cannabinoid receptors: CB1 and CB2. CB1 receptors are mainly found in the brain, peripheral nerves, and autonomic nervous system,[5] whereas CB2 receptors are found both in the neurons and immune cells.[6] THC exerts its effects primarily via CB1 receptors.

The Laws Regarding Cannabis

In the United States, cannabis is an illicit drug either to possess or trade. Since the inception of the Controlled Substance Act in 1970, the US Federal Law penalizes any act of possessing, dispensing, and prescribing marijuana. Enforcement of prohibition carries an annual price tag of up to $7.7 billion in the United States alone.[7] However, since 1996 the situation has been changing rapidly—14 states (California, Alaska, Oregon, Washington, Maine, Hawaii, Colorado, Nevada, Vermont, Montana, Rhode Island, New Mexico, Michigan, and New Jersey) already have amended their state laws to allow the use of marijuana by persons with debilitating medical conditions as certified by licensed physicians.[8,9] The impact has been significant: a recent study in Washington estimated that per annum, up to 2000 licensed physicians have prescribed medical cannabis;[10] in California, more than 350,000 patients already possess a physician's recommendation to use cannabis.[11] Nevertheless, among these 14 states, there is substantial variation in the regulation of the quality control, prescription limit, patient registry, and dispensing outlets. For example, in Oregon and Washington, it is legal to possess up to 24 ounces of marijuana, but in Nevada, Montana, and Alaska, the legal limit is only 1 ounce.[8] Cannabis is currently schedule I; additional research would be facilitated if the drug were reclassified to schedule II.[8] From a public health standpoint, there is some evidence that decriminalization of cannabis could free up law enforcement resources to curtail other trafficking activities without leading to increased cannabis abuses.[12] Overall, however, the US Federal law remains unchanged regarding the penal stance toward marijuana, creating various ambiguities and difficulties. For those veterans who are permitted to use medical marijuana by law of their state, these difficulties have been lessened. This has posed an administrative dilemma for those veterans who are allowed to use; the Department of Veterans Affairs issued a directive in July 2010 that permits veterans to continue their use of medical marijuana in states where it is legal without losing their medical benefits from Veterans Affairs.[13]

Recent news from USA Today [14] reports that the US federal government has issued warning letters to several states that have approved the use of medical marijuana with an implication that anyone involved in the growth, operation, or legal regulation of medical marijuana will be subjected to prosecution. These states include Washington, California, Montana, and Rode Island. This was coupled by recent large-scale raids at marijuana growing operations in Montana. Despite reassurance from Eric Holder, US Attorney General, that the penal policy is directed at those who violate both deferral and state laws, this unexpected siren from the federal government has been heard loud and clear, leading Governor Chris Gregoire, of the state of Washington, to abort a proposal to create licensed marijuana dispensaries and Governor Chris Christie, of the state of New Jersey, to postpone plans for marijuana operators.

In Canada, it is also illegal to trade or possess 104 marijuana according to provincial and government laws. However, access to marijuana for medical use is possible under Health Canada's Marijuana Medical Access Regulations, which came into force on July 30, 2001.[14] The regulations clearly outline 2 categories of persons who can apply to possess for an authorization to possess marijuana for medical purposes. Category 1 refers to people with end-of-life care; seizures from epilepsy; severe pain and/or persistent muscle spasms caused by multiple sclerosis, spinal cord diseases, or spinal cord injury; severe pain; cachexia; anorexia; weight loss and/or severe nausea from cancer or HIV/AIDS infection. A medical declaration from a licensed medical practitioner is required. Category 2 refers to people who have debilitating symptom(s) of medical condition(s), other than those described in category 1, which have failed conventional medical treatment. An assessment by a designated specialist is necessary along with a medical declaration from a licensed medical practitioner.

Under the regulations, the maximum amount of marijuana that can be possessed by any authorized user is a 30-day total of daily requirement. Health Canada sources its supply of dried marijuana and seeds from Prairie Plant Systems Incorporated (Saskatoon, Saskatchewan, Canada), a company that specializes in the growing, harvesting, and processing of plants for pharmaceutical products and research. Alternatively, authorized marijuana users can apply for a permit to produce and grow their own supply provided they meet specific and detailed criteria.

The Harms of Cannabis

Physical and Psychiatric Effects

Among naive users, cannabis smoking often leads to adverse effects. Physical symptoms include increased heart rate and fluctuations in blood pressure;[15] psychomotor sequelae include euphoria, anxiety, psychomotor retardation, and impairment of cognition and memory.[16] The estimated lethal dose for humans is between 15 g and 70 g.[3] When compared with cigarette smoke, cannabis contains a similar array of detrimental and carcinogenic compounds, some of which are present even at higher concentrations.[17] Among chronic users, population studies have associated cannabis use with decreased pulmonary function, chronic obstructive airway diseases, and pulmonary infections,[18] although data may be confounded by concomitant tobacco smoking and other social factors. In vitro and in vivo animal studies have demonstrated mutagenic effects of cannabis smoke, and precancerous pulmonary pathology as seen in tobacco smokers has been described in cannabis users.[19] Nevertheless, there is still inconsistency from the published literature regarding an increased risk for upper respiratory tract cancer caused by cannabis smoking.[3,18] Various reports have associated cannabis with cardiac arrhythmias,[20,21] coronary insufficiency[22–24]and myocardial infarction.[25,26] A retrospective cross-sectional study revealed a 4.8-times increased risk of developing myocardial infarction within the first hour after smoking cannabis. Earlier data from population studies[27,28] and meta-analysis[29] have associated cannabis smoking with low birth weight,[29] which is maybe confounded by cigarette smoking and socioeconomic status and is not supported by more recent studies.[30,31] Finally, the controversial link of cannabis use and psychosis has found more support in recent publications.[32–34]

Dependence and Abuse

Cannabis is recognized as a substance with a high potential for dependence, which occurs in 1 out of 10 people who have ever used cannabis. It leads to behaviors of preoccupation, compulsion, reinforcement, and withdrawal after chronic use.[35] An Australian survey found that symptoms of cannabis withdrawal satisfied the diagnostic criteria of both International Classification of Diseases 10 and Diagnostic and Statistical Manual of Mental Disorders IV for substance dependence, which included sleep disturbance, anorexia, irritability, dysphoria, lethargy, and cravings.[36] In the United States, cannabis is now ranked among alcohol and tobacco as one of the most common substances of among adolescents.[37] There is also ample evidence indicating that regular use of cannabis predicts subsequent psychosocial problems and abuse behavior of other addictive substances. A review of cohort studies by McLaren et al[38] supported a causal link between cannabis use and psychosis. A recent 10-year follow-up study of adolescents in Australia who used cannabis occasionally were found to be at higher risks of drug abuse and educational problems.[39] However, several issues have been identified in the published literature about cannabis, which have limited our understanding on the adverse effects of cannabis: (1) lack of consensus on the definition and classification of different types of cannabis users (heavy, regular, occasional, and nonusers); (2) variable quality of studies regarding design, effect sizes, and control of confounding factors; and (3) the polarization of the approach to either studying nonusers versus light/infrequent users or, infrequent/light/nondependent users versus frequent/heavy/dependent users.[40]

New Kids on the Block

Recently, synthetic analogues of marijuana, known generically as "spice" or "K2," have gained rapid popularity among youths in the Unites States and Europe. Marketed as an incense or herbal blend, the exact constituents of spice has been a myth, and its place of origin is often unclear. Despite sharing similar psychotropic effects as genuine cannabis, spice cannot be reliably tested by drug screens and poses a technical problem for the law enforcement; hence it is capable of evading legal scrutiny among most states in America. A report from the Drug Enforcement Administration of the US Department of Justice in June 2010 had divulged the possible constituents of spice (or K2), which included HU-210, JWH-018, JWH-073 and CP-47,497,[41] all of which were synthetic cannabinoids legally endorsed for scientific research. This was echoed by a recent research publication that identified a synthetic cannabinoid in commercially obtained spice, JWH-018, which activated CB1.[42]

Analgesic Potential and Synergism With Opioids

Despite legal curtailment, cannabis is still used by 10% to 15% of patients with multiple sclerosis[43] and noncancer types of chronic pain[44] for both analgesia and psychological detachment. Various well-designed, randomized, placebo-controlled trials have shown that smoked cannabis can relieve peripheral,[45] posttraumatic,[46] and HIV-induced[47,48] neuropathic pain. Evidence has been accumulating from molecular and cell-signaling studies that suggest that the opioids and cannabinoid systems can interact synergistically to enhance analgesic effects.[49] Animal studies have shown that topical cannabinoid enhances the action of topical morphine,[50] an effect that is preserved in a morphine-tolerant state.[51] Moreover, cannabinoids are increasingly being recognized in animal models for their potential sparing effects with opioids[52] of neuropathic pain and arthritic pain.[53] Although similar effects have not been translated to human studies, Robert et al[54] found a synergistic affective analgesia between Δ9-THC and morphine in experimentally induced pain in human volunteers.

Evidence From Clinical Studies

To review the latest evidence of cannabis use and its derivatives, a literature search was conducted from the MEDLINE, EMBASE, PsycINFO, and Cochrane Database of Systematic Reviews from their inception dates to 30 November 2010, using the following keywords: "cannabis," "marijuana," "Δ9-tetrahydrocannabinol," "clinical trial," "benefits," and "side effects." Relevant articles were selected and their quality of evidence was rated according to the Strength of Recommendations Taxonomy (SORT),[56] with recommendations rated as A, B, or C. The results are summarized in Table 1. In brief, the efficacy of smoked cannabis has been studied for Gilles de la Tourette syndrome, glaucoma, and pain, with good evidence for clinical benefits in HIV-induced neuropathic pain. Oral extract of cannabis has better evidence of relieving self-reported symptoms of spasticity caused by multiple sclerosis. Finally, the oromucosal form of cannabis extract (Sativex, GW Pharmaceuticals) is efficacious for peripheral and central neuropathic pain, especially that caused by multiple sclerosis.

Table 1. Clinical Studies of Cannabis and Its Derivatives with SORT Level of Recommendation56

AgentCondition IndicatedForm of deliveryNature of StudyPatients (n)Outcome MeasuresOutcomeSORT Level of RecommendationReference
Cannabis Gilles de la Tourette Syndrome Smoking Case report 3 Self-reported frequency of motor tics 50% to 70% remission C Sandyk et al57
Cannabis Gilles de la Tourette Syndrome Smoking Case report 1 Self-reported symptoms 100% remission C Hemming et al58
Cannabis Glaucoma Smoking single dose Double-blinded cross-over placebo-controlled RCT 18 Intraocular pressure Significant reduction B Merritt et al59
Cannabis Neuropathic pain in HIV patient Smoking 5 days a week for 2 weeks Prospective placebo-controlled RCT 28 Pain intensity using Descriptor Differential Scale Improvement in pain (P = .016) A Ellis et al49
Cannabis Sensory neuropathic pain in HIV patient Smoking 3 times a day for 5 days Double-blinded cross-over placebo-controlled RCT 50 Chronic pain ratings Reduction of pain by 34% (P = .03) A Abrams et al48
Cannabis Capsaicin-induced pain in volunteers Smoking single dose at various concentrations Double-blinded cross-over placebo-controlled RCT 15 Pain scores and McGill Pain Questionnaire Pain reduction at medium dose within a certain time frame only B Wallace et al60
Cannabis Acute inflammatory pain in volunteers Single oral dose of encapsulate extract Double-blinded cross-over placebo-controlled RCT 18 Threshold to heat and electricity in areas with UV-induced sunburnt No effect on pain thresholds B Kraft et al61
Cannabis Spasticity due to multiple sclerosis Escalating dose of oral encapsulate extract Double-blinded cross-over placebo-controlled RCT 50 Spasms frequency and mobility Improvement in spasms frequency (P= .013) and mobility (P = .01) A Vaney et al62
Cannabis Spasticity caused by multiple sclerosis Titrating oral dose of cannabis extract Double-blinded placebo-controlled RCT 327 Ashworth score and self-reported spasticity Improvement of self-report ratings of pain and spasticity (P= .003) A Zajicek et al63
Δ9-THC Gilles de la Tourette Syndrome Single oral dose Cross-over placebo-controlled RCT 12 TSSL, STSS, YGTSS scores Significant reduction in TSSL score (P = .015), nil for STSS and YGTSS A Müller-Vahl et al64
Δ9-THC Gilles de la Tourette Syndrome Daily oral dose for 6 weeks Placebo-controlled RCT 24 TSSL,TS-CGI, STSS; YGTSS Significant reduction in TSSL score using ANOVA (P = .037), nil for TS-CGI, STSS, YGTSS A Müller-Vahl et al65
Δ9-THC Spasticity caused by multiple sclerosis Escalating dose for 5 days Double-blinded cross-over placebo-controlled RCT 13 Subjective rating and objective measure of spasticity Significant in both scores A Ungerleider et al66
Δ9-THC Spasticity due to multiple sclerosis Titrating oral dose of Δ9-THC Double-blinded placebo-controlled RCT 330 Ashworth score and self-reported spasticity Improvement of self-report ratings of pain and spasticity (P= .003) A Zajicek et al63
Δ9-THC Postoperative pain Single oral dose on postoperative day 2 Double-blinded placebo-controlled RCT 40 Summed pain intensity difference 6 hours after administration No significant difference B Buggy et al67
Δ9-THC Refractory neuropathic pain Titrating oral dose Open label pilot 8 Neuropathic pain score and quality of life No apparent effect C Attal et al68
Δ9-THC Glioblastoma multiforme Daily intracranial tumour injection up to 64 days Phase I cohort pilot study 9 Safety of intracranial route of administration Intracranial route seems to be safe and may slow down tumour growth C Guzman et al69
Dronabinol (synthetic Δ9-THC) Alzheimer's disease Twice-daily oral dose for 6 weeks Double-blinded cross-over placebo-controlled RCT 15 Body weight, triceps skin fold, disturbed behavior, affect A trend of improvement reported but no significance quoted B Volicer et al70
Dronabinol (synthetic Δ9-THC) Alzheimer's disease Daily oral dose for 2 weeks Open label pilot 6 Nocturnal motor activity score and Neuropsychiatric Inventory Significant improvement in both (P = .028 and P = 0027) C Walther et al71
Dronabinol (synthetic Δ9-THC) Anorexia and weight loss in AIDS Twice-daily oral dose for 6 weeks Placebo-controlled RCT 139 VAS for appetite, mood, and nausea Significant change in appetite (38%; P = .015); mood (10%; P = .06); and nausea (20%; P = .05) A Beal et al72
Nabilone Spasticity caused by spinal cord injury Twice-daily oral dose for 4 weeks Double-blinded cross-over placebo-controlled RCT 12 Ashworth Scale, Total Ashworth Score Significant reduction, P= .003 and 0.001 respectively A Pooyania et al73
Nabilone Pain caused by fibromyalgia Oral dose for 4 weeks Double-blinded placebo-controlled RCT 40 VAS and Fibromyalgia impact questionnaire Significant reduction in both scores (P < .02) A Skrabek et al74
Sativex (extract of cannabis containing Δ9-THC and cannabidiol) Peripheral neuropathic pain Self-titrating dose of oromucosal spray for 5 weeks Double-blinded placebo-controlled RCT 125 Various pain intensity scores Significant reduction, (P= .001 to P= .04) A Nurmikko et al75
Sativex (extract of cannabis containing Δ9-THC and cannabidiol) Intractable neurogenic symptoms Self-titrating dose of oromucosal spray for 2 weeks Double-blinded cross-over placebo-controlled RCT 20 Self-report symptoms and adverse effects Significant relief in pain with certain domains reaching significance of P < .05 A Wade et al76
Sativex (extract of cannabis containing Δ9-THC and cannabidiol) Central pain in multiple sclerosis Self-titrating dose of oromucosal spray for 4 weeks Double-blinded placebo-controlled RCT 66 11-point scale for pain and sleep disturbance Significant reduction of pain (P = .005) and sleep disturbance (P = .003) A Rog et al77
Sativex (extract of cannabis containing Δ9-THC and cannabidiol) Bladder dysfunction in multiple sclerosis Single daily dose for 8 weeks Open label pilot study 15 Occurrence of urinary incontinence, frequency, nocturia Significant reduction in all 3 domains (P< .05) A Brady et al78

RCT, randomized controlled trial; UV, ultraviolet; TSSL,; STSS,; YGTSS,; TS-CGI, ANOVA, analysis of variance; VAS, Visual Analog Scale; THC, tetrahydrolcannabinol.

The Challenges of using Cannabis

Despite the evidence of benefits in certain conditions, the use of medical marijuana within a legal jurisdiction still faces a number of challenges:

  • Method of Delivery and Quality Control. Smoking raw cannabis remains the most common and easiest route of delivery, but the actual amount of cannabinoids deliverable to the alveolar space varies considerably depending on the individual's techniques of inhalation/exhalation, the percentage of aeroingestion, and the individual's functional lung capacity. Without prior training, it could be difficult for a family physician in daily practice to advise an eligible patient on the proper techniques of administration and quality control of prescription regarding medical marijuana. The content of THC in cannabis may vary remarkably according by geographic origin,[56] the parts of plant being used (buds versus stem and seeds), the methods of storage, and the techniques of cultivation.[79] There are 2 main strains used in medical marijuana: the Sativa and the Indica. The Sativa plant is usually taller with longer leaves that grow better outdoors, whereas the Indica plant is more bushy with shorter leaves that thrive better indoors. Although both strains exist in pure forms, various combinations of the 2 strains are packaged as medical marijuana, which may result in variable therapeutic and side effects. Health Canada's policy of adopting a centralized source of medical marijuana from an approved plantation is a good way to assure quality; however, it is still technically difficult to endorse it globally for all licensed users and growers. As a prescription, standardization and titration of dose efficacy remain a challenge for medical marijuana.
  • Adequate Monitoring and Prevention of Addiction. As with other substances of abuse, cannabis may lead to varying adverse effects and addiction potential among different individuals. Before facilitating an eligible person to receive medical marijuana, family physicians should possess the knowledge and skills to screen for addiction potential. During the course of treatment, close surveillance of the patient to prevent addiction and adverse effects, in collaboration with a specialist when necessary, remains a top priority. In Canada and in those American states where it is legal to use medical marijuana, more training and educational resources should be made available for the practicing family physician to enhance their competence in approaching cannabis.
  • Contaminants in Cannabis. Studies have reported an alarming level of biological contaminants in cannabis, which includeAspergillus fungus[80,81] and bacteria,[82] potentially leading to fulminant pneumonia, especially among the immunosuppressed.[83] Nonbiological contaminants also have been found, which include heavy metals from soil like aluminum[84] and cadmium, the latter of which seems to be absorbed by the cannabis plant in particularly high concentrations.[85] Organophosphate pesticides are other nonbiological contaminants that are found less in cannabis cultivated outdoors than indoors.[36] Finally, tiny glass beads or sand have been found in street samples of cannabis, which were added for weight to boost profits and can cause damage to the oral mucosa and lungs.[86]
  • Contamination by Cannabis. Secondary inhalation of cannabis fumes released by primary smokers is a theoretical but negligible threat, as shown by a study of airborne particulates in urban Spain[87] and another study of passive exposure to cannabis smoke in a Netherlands coffee shop.[88] More research in this area is warranted from the perspective of public health.

The Controversy Remains

In 1969, an article published in the New England Journal of Medicine quoted from the Wootton Report that cannabis is "a potent drug, having as wide a capacity as alcohol to alter mood, judgment, and functional ability, and admitted that it is a dangerous drug in that sense, but in terms of physical harmfulness much less dangerous than opiates, amphetamines, and barbiturates and also less dangerous than alcohol."[89] Since then, scientific and clinical data have helped us understand the mechanisms of actions of cannabis and its derived compounds for treating chronic and neuropathic pain, highlighting the potential analgesic synergism with opioids and the potential of an opiate sparing effect in clinical settings. In particular, animal studies have recently shown that cannabidiol (CBD), a nonpsychoactive constituent of marijuana, is capable of decreasing self-administration and drug-seeking behavior caused by heroin,[90] in addition to other anti-inflammatory antipsychotic and neuroprotective effects.[91,92] Another observational study of the ratio of CBD:THC from street cannabis samples suggests that a higher CBD content reduced reinforcing behavior and attention bias to marijuana. Further directions of research include a better understanding of the mechanisms of action of CBD and its interplay with THC, plus bioengineering a safer marijuana strain that contains the appropriate composition of CBD and THC for optimal therapeutic effects with the least adverse profile and addictive potential. Thus, important issues of dosage standardization, quality control, adverse effects profiling, and prevention of addiction could be resolved. Until then, family physicians in North America and Canada continue to face the under-reported use of cannabis in our society and its risks of abuse.

References

  1. Zuardi AW. History of cannabis as a medicine: a review. Rev Bras Psiquiatr 2006;28(2):153–7.
  2. Touw M. The religious and medicinal uses of Cannabis in China, India and Tibet. J Psychoactive Drugs 1981;13(1):23–34.
  3. Hall W, Degenhardt L. Adverse health effects of non-medical cannabis use. Lancet 2009;374(9698):1383–91.
  4. Ashton CH. Pharmacology and effects of cannabis: a brief review. Br J Psychiatry 2001;178:101–6.
  5. Tsou K, Brown S, Sanudo-Pena MC, Mackie K, Walker JM. Immunohistochemical distribution of cannabinoid CB1 receptors in the rat central nervous system. Neuroscience 1998;83(2):393–411.
  6. Galiegue S, Mary S, Marchand J, et al. Expression of central and peripheral cannabinoid receptors in human immune tissues and leukocyte subpopulations. Eur J Biochem 1995;232(1):54–61.
  7. Egan D, Miron JA. The budgetary implications of marihuana prohibition. In: Earleywine M, ed. Pot Politics: Marijuana and the Costs of Prohibition. New York: Oxford University Press;2007:17–39.
  8. Hoffmann DE, Weber E. Medical marijuana and the law. N Engl J Med 2010;362(16):1453–7.
  9. MacDonald J. Medical marijuana: informational resources for family physicians. Am Fam Physician 2009;80(8):779.
  10. Aggarwal SK, Kyashna-Tocha M, Carter GT. Dosing medical marijuana: rational guidelines on trial in Washington State. Med Gen Med 2007;9(3):52.
  11. O'Connell TJ, Bou-Matar CB. Long term marijuana users seeking medical cannabis in California (2001–2007): demographics, social characteristics, patterns of cannabis and other drug use of 4117 applicants. Harm Reduct J 2007;4:16.
  12. Single E, Christie P, Ali R. The impact of cannabis decriminalisation in Australia and the United States. J Public Health Policy 2000;21(2):157–86.
  13. US Department of Veterans Affairs. Medical marijuana. VHA Directive 2010–035. Washington DC: Veterans Health Administration; July 2010. Available at: http://www.va.gov/vhapublications/ViewPublication.asp?pub_ID=2276. Accessed.
  14. Lifland JB. States reassess medical marijuana laws after warnings. 3 May 2011. Available at: http://yourlife.usatoday.com/health/medical/story/2011/05/States-reassess-medical-marijuana-laws-after-warnings/46756064/1#uslPageReturn>. Accessed 31 May 2011.
  15. Health Canada. Fact sheet–medical access to marihuana. 2008. Available at: http://www.hc-sc.gc.ca/dhp-mps/marihuana/law-loi/fact_sheet-infofiche-eng.php. Accessed 31 May 2011.
  16. Hall W, Solowij N. Adverse effects of cannabis. Lancet 1998;352(9140):1611–6.
  17. Grotenhermen F. The toxicology of cannabis and cannabis prohibition. Chem Biodivers 2007;4(8):1744–69.
  18. Moir D, Rickert WS, Levasseur G, et al. A comparison of mainstream and sidestream marijuana and tobacco cigarette smoke produced under two machine smoking conditions. Chem Res Toxicol 2008;21(2):494–502.
  19. Reid PT, Macleod J, Robertson JR. Cannabis and the lung. J R Coll Physicians Edinb 2010;40(4):328–34.
  20. Tashkin DP. Smoked marijuana as a cause of lung injury. Monaldi Arch Chest Dis 2005;63(2):93–100.
  21. Lehavi A, Shay M, Gilony C, Even L. [Marijuana smoking and paroxysmal atrial fibrillation]. Harefuah 2005;144(1):2–3, 72.
  22. Baranchuk A, Johri AM, Simpson CS, Methot M, Redfearn DP. Ventricular fibrillation triggered by marijuana use in a patient with ischemic cardiomyopathy: a case report. Cases J 2008;1(1):373.
  23. Basnet S, Mander G, Nicolas R. Coronary vasospasm in an adolescent resulting from marijuana use. Pediatr Cardiol 2009;30(4):543–5.
  24. Bailly C, Merceron O, Hammoudi N, Dorent R, Michel PL. Cannabis induced acute coronary syndrome in a young female. Int J Cardiol 2010;143(1): e4–6.
  25. Dwivedi S, Kumar V, Aggarwal A. Cannabis smoking and acute coronary syndrome: two illustrative cases. Int J Cardiol 2008;128(2): e54–7.
  26. Kocabay G, Yildiz M, Duran NE, Ozkan M. Acute inferior myocardial infarction due to cannabis smoking in a young man. J Cardiovasc Med (Hagerstown) 2009;10(9):669–70.
  27. Cappelli F, Lazzeri C, Gensini GF, Valente S. Cannabis: a trigger for acute myocardial infarction? A case report. J Cardiovasc Med (Hagerstown) 2008;9(7):725–8.
  28. Fergusson DM, Horwood LJ, Northstone K. Maternal use of cannabis and pregnancy outcome. BJOG 2002;109(1):21–7.
  29. Zuckerman B, Frank DA, Hingson R, et al. Effects of maternal marijuana and cocaine use on fetal growth. N Engl J Med 1989;320(12):762–8.
  30. English DR, Hulse GK, Milne E, Holman CD, Bower CI. Maternal cannabis use and birth weight: a meta-analysis. Addiction 1997;92(11):1553–60.
  31. van Gelder MM, Reefhuis J, Caton AR, Werler MM, Druschel CM, Roeleveld N. Characteristics of pregnant illicit drug users and associations between cannabis use and perinatal outcome in a population-based study. Drug Alcohol Depend 2010;109(1–3):243–7.
  32. Schempf AH, Strobino DM. Illicit drug use and adverse birth outcomes: is it drugs or context? J Urban Health 2008;85(6):858–73.
  33. Le Bec PY, Fatseas M, Denis C, Lavie E, Auriacombe M. [Cannabis and psychosis: search of a causal link through a critical and systematic review]. Encephale 2009;35(4):377–85.
  34. Shapiro GK, Buckley-Hunter L. What every adolescent needs to know: cannabis can cause psychosis. J Psychosom Res 2010;69(6):533–9.
  35. Dragt S, Nieman DH, Becker HE, et al. Age of onset of cannabis use is associated with age of onset of high-risk symptoms for psychosis. Can J Psychiatry 2010;55(3):165–71.
  36. Miller NS, Gold MS. The diagnosis of marijuana (cannabis) dependence. J Subst Abuse Treat 1989;6(3):183–92.
  37. Swift W, Gates P, Dillon P. Survey of Australians using cannabis for medical purposes. Harm Reduct J 2005;2:18.
  38. Latimer W, Zur J. Epidemiologic trends of adolescent use of alcohol, tobacco, and other drugs. Child Adolesc Psychiatr Clin N Am 2010;19(3):451–64.
  39. McLaren JA, Silins E, Hutchinson D, Mattick RP, Hall W. Assessing evidence for a causal link between cannabis and psychosis: a review of cohort studies. Int J Drug Policy 2010;21(1):10–9.
  40. Degenhardt L, Coffey C, Carlin JB, Swift W, Moore E, Patton GC. Outcomes of occasional cannabis use in adolescence:10-year follow-up study in Victoria, Australia. Br J Psychiatry 2010;196:290–5.
  41. Temple EC, Brown RF, Hine DW. The 'grass ceiling': limitations in the literature hinder our understanding of cannabis use and its consequences. Addiction 2011;106(2):238–44.
  42. Drug Enforcement Adminstration, US Department of Justice. Herbal high-legal synthetic cannabinoids produce illicit marijuana effects: spice, k2 and other "legal" products sold as marijuana alternatives. June 2010. Available at: www.wyptac.org/upload/DEA%20K2%20Fact%20Sheet.pdf. Accessed 31 May 2011.
  43. Atwood BK, Huffman J, Straiker A, Mackie K. JWH018, a common constituent of 'Spice' herbal blends, is a potent and efficacious cannabinoid CB receptor agonist. Br J Pharmacol 2010;160(3):585–93.
  44. Clark AJ, Ware MA, Yazer E, Murray TJ, Lynch ME. Patterns of cannabis use among patients with multiple sclerosis. Neurology 2004;62(11):2098–100.
  45. Ware MA, Doyle CR, Woods R, Lynch ME, Clark AJ. Cannabis use for chronic non-cancer pain: results of a prospective survey. Pain 2003;102(1–2):211–6.
  46. Wilsey B, Marcotte T, Tsodikov A, et al. A randomized, placebo-controlled, crossover trial of cannabis cigarettes in neuropathic pain. J Pain 2008;9(6):506–21.
  47. Ware MA, Wang T, Shapiro S, et al. Smoked cannabis for chronic neuropathic pain: a randomized controlled trial. CMAJ 2010;182(14): E694–701.
  48. Abrams DI, Jay CA, Shade SB, et al. Cannabis in painful HIV-associated sensory neuropathy: a randomized placebo-controlled trial. Neurology 2007;68(7):515–21.
  49. Ellis RJ, Toperoff W, Vaida F, et al. Smoked medicinal cannabis for neuropathic pain in HIV: a randomized, crossover clinical trial. Neuropsychopharmacology 2009;34(3):672–80.
  50. Bushlin I, Rozenfeld R, Devi LA. Cannabinoid-opioid interactions during neuropathic pain and analgesia. Curr Opin Pharmacol 2010;10(1):80–6.
  51. Yesilyurt O, Dogrul A, Gul H, et al. Topical cannabinoid enhances topical morphine antinociception. Pain 2003;105(1–2):303–8.
  52. Yesilyurt O, Dogrul A. Lack of cross-tolerance to the antinociceptive effects of systemic and topical cannabinoids in morphine-tolerant mice. Neurosci Lett 2004;71(2–3):122–7.
  53. Elikkottil J, Gupta P, Gupta K. The analgesic potential of cannabinoids. J Opioid Manag 2009;5(6):341–57.
  54. Cox ML, Haller VL, Welch SP. Synergy between delta9-tetrahydrocannabinol and morphine in the arthritic rat. Eur J Pharmacol 2007;567(1–2):125–30.
  55. Roberts JD, Gennings C, Shih M. Synergistic affective analgesic interaction between delta-9-tetrahydrocannabinol and morphine. Eur J Pharmacol 2006;530(1–2):54–8.
  56. Ebell MH, Siwek J, Weiss BD, et al. Strength of recommendation taxonomy (SORT): a patient-centered approach to grading evidence in the medical literature. J Am Board Fam Pract 2004;17(1):59–67.
  57. Sandyk R, Awerbuch G. Marijuana and Tourette's syndrome. J Clin Psychopharmacol 1998;8(6):444–5.
  58. Hemming M, Yellowlees PM. Effective treatment of Tourette's syndrome with marijuana. J Psychopharmacol 1993;7(4):389–91.
  59. Merritt JC, Crawford WJ, Alexander PC, Anduze AL, Gelbart SS. Effect of marihuana on intraocular and blood pressure in glaucoma. Ophthalmology 1980;87(3):222–8.
  60. Wallace M, Schulteis G, Atkinson JH, et al. Dose-dependent effects of smoked cannabis on capsaicin-induced pain and hyperalgesia in healthy volunteers. Anesthesiology 2007;107(5):785–96.
  61. Kraft B, Frickey NA, Kaufmann RM, et al. Lack of analgesia by oral standardized cannabis extract on acute inflammatory pain and hyperalgesia in volunteers. Anesthesiology 2008;109(1):101–10.
  62. Vaney C, Heinzel-Gutenbrunner M, Jobin P, et al. Efficacy, safety and tolerability of an orally administered cannabis extract in the treatment of spasticity in patients with multiple sclerosis: a randomized, double-blind, placebo-controlled, crossover study. Mult Scler 2004;10(4):417–24.
  63. Zajicek JP, Sanders HP, Wright DE, et al. Cannabinoids in multiple sclerosis (CAMS) study: Safety and efficacy data for 12 months follow up. J Neurol Neurosurg Psychiatry 2005;76(12):1664–9.
  64. Muller-Vahl KR, Schneider U, Koblenz A, et al. Treatment of Tourette's syndrome with 9-tetrahydrocannabinol (THC): a randomized crossover trial. Pharmacopsychiatry 2002;35(2):57–61.
  65. Muller-Vahl KR, Schneider U, Prevedel H, et al. 9-tetrahydrocannabinol (THC) is effective in the treatment of tics in Tourette syndrome: a 6-week randomized trial. J Clin Psychiatry 2003;64(4):459–65.
  66. Ungerleider JT, Andyrsiak T, Fairbanks L, Ellison GW, Myers LW. Delta-9-THC in the treatment of spasticity associated with multiple sclerosis. Adv Alcohol Subst Abuse 1987;7(1):39–50.
  67. Buggy DJ, Toogood L, Maric S, Sharpe P, Lambert DG, Rowbotham DJ. Lack of analgesic efficacy of oral -9-tetrahydrocannabinol in postoperative pain. Pain 2003;106(1–2):169–72.
  68. Attal N, Brasseur L, Guirimand D, Clermond-Gnamien S, Atlami S, Bouhassira D. Are oral cannabinoids safe and effective in refractory neuropathic pain? Eur J Pain 2004;8(2):173–7.
  69. Guzman M, Duarte MJ, Blazquez C, et al. A pilot clinical study of 9-tetrahydrocannabinol in patients with recurrent glioblastoma multiforme. Br J Cancer 2006;95(2):197–203.
  70. Volicer L, Stelly M, Morris J, McLaughlin J, Volicer BJ. Effects of dronabinol on anorexia and disturbed behavior in patients with Alzheimer's disease. Int J Geriatr Psychiatry 1997;12(9):913–9.
  71. Walther S, Mahlberg R, Eichmann U, Kunz D. Delta-9-tetrahydrocannabinol for nighttime agitation in severe dementia. Psychopharmacology (Berl) 2006;185(4):524–8.
  72. Beal JE, Olson R, Laubenstein L, et al. Dronabinol as a treatment for anorexia associated with weight loss in patients with AIDS. J Pain Symptom Manage 1995;10(2):89–97.
  73. Pooyania S, Ethans K, Szturm T, Casey A, Perry D. A randomized, double-blinded, crossover pilot study assessing the effect of nabilone on spasticity in persons with spinal cord injury. Arch Phys Med Rehabil 2010;91(5):703–7.
  74. Skrabek RQ, Galimova L, Ethans K, Perry D. Nabilone for the treatment of pain in fibromyalgia. J Pain 2008;9(2):164–73.
  75. Nurmikko TJ, Serpell MG, Hoggart B, Toomey PJ, Morlion BJ, Haines D. Sativex successfully treats neuropathic pain characterised by allodynia: a randomised, double-blind, placebo-controlled clinical trial. Pain 2007;133(1–3):210–20.
  76. Wade DT, Robson P, House H, Makela P, Aram J. A preliminary controlled study to determine whether whole-plant cannabis extracts can improve intractable neurogenic symptoms. Clin Rehabil 2003;17(1):21–9.
  77. Rog DJ, Nurmikko TJ, Friede T, Young CA. Randomized, controlled trial of cannabis-based medicine in central pain in multiple sclerosis. Neurology 2005;65(6):812–9.
  78. Brady CM, DasGupta R, Dalton C, Wiseman OJ, Berkley KJ, Fowler CJ. An open-label pilot study of cannabis-based extracts for bladder dysfunction in advanced multiple sclerosis. Multiple Sclerosis 2004;10(4):425–33.
  79. Bocker KB, Gerritsen J, Hunault CC, Kruidenier M, Mensinga TT, Kenemans JL. Cannabis with high Delta(9)-THC contents affects perception and visual selective attention acutely: an event-related potential study. Pharmacol Biochem Behav 2010;96(1):67–74.
  80. McLaren J, Swift W, Dillon P, Allsop S. Cannabis potency and contamination: a review of the literature. Addiction 2008;103(7):1100–9.
  81. Kagen SL, Kurup VP, Sohnle PG, Fink JN. Marijuana smoking and fungal sensitization. J Allergy Clin Immunol 1983;71(4):389–93.
  82. Kurup VP, Resnick A, Kagen SL, Cohen SH, Fink JN. Allergenic fungi and actinomycetes in smoking materials and their health implications. Mycopathologia 1983;82(1):61–4.
  83. Ungerleider JT, Andrysiak T, Tashkin DP, Gale RP. Contamination of marihuana cigarettes with pathogenic bacteria–possible source of infection in cancer patients. Cancer Treat Rep 1982;66(3):589–91.
  84. Chusid MJ, Gelfand JA, Nutter C, Fauci AS. Pulmonary aspergillosis, inhalation of contaminated marijuana smoke, chronic granulomatous disease. Ann Intern Med 1975;82(5):682–3.
  85. Exley C, Begum A, Woolley MP, Bloor RN. Aluminum in tobacco and cannabis and smoking-related disease. Am J Med 2006;119(3):276.e9–211.
  86. Shi G, Cai Q. Cadmium tolerance and accumulation in eight potential energy crops. Biotechnol Adv 2009;27(5):555–61.
  87. Delourme J, Delattre C, Godard P, Steenhouwer F, Just N. [Respiratory consequences of inhalation of adulterated cannabis]. Rev Mal Respir 2009;26(5):552–6.
  88. Viana M, Querol X, Alastuey A, et al. Drugs of abuse in airborne particulates in urban environments. Environ Int 2010;36(6):527–34.
  89. Rohrich J, Schimmel I, Zorntlein S, et al. Concentrations of delta9-tetrahydrocannabinol and 11-nor-9-carboxytetrahydrocannabinol in blood and urine after passive exposure to Cannabis smoke in a coffee shop. J Anal Toxicol 2010;34(4):196–203.
  90. Lister J. Cannabis controversy and other sundry troubles. N Engl J Med 1969;280(13):712–4.
  91. Ren Y, Whittard J, Higuera-Matas A, Morris CV, Hurd YL. Cannabidiol, a nonpsychotropic component of cannabis, inhibits cue-induced heroin seeking and normalizes discrete mesolimbic neuronal disturbances. J Neurosci 2009;29(47):14764–9.
  92. Zuardi AW. Cannabidiol: from an inactive cannabinoid to a drug with wide spectrum of action. Rev Bras Psiquiatr 2008;30(3):271–80.
  93. Zuardi AW, Crippa JA, Hallak JE, Moreira FA, Guimaraes FS. Cannabidiol, a Cannabis sativa constituent, as an antipsychotic drug. Braz J Med Biol Res 2006;39(4):421–9

J Am Board Fam Med. 2011;24(4):452-462. © 2011 American Board of Family Medicine

De-escalating the Toxicologically Altered Aggressive Individual

Toxicologists are often faced with cases where the subject in question had altered mental status as a consequence of the effects of illicit substances.  In this setting, dealing with aggressive patients can make a big difference in outcome. Patient death or injury resulting from the use of restraint and seclusion is an increasing concern in the field and in prison. Excessive and inappropriate TASER use has also been associated with sudden death.  A well-known 1998 article[1] documented 142 restraint-related deaths nationwide over a decade, 40% of which were attributed to unintentional asphyxiation during restraint. Restraint not only poses a risk for patient harm but also is physically and emotionally traumatizing for staff involved in the incident. Stefan pointed out that "high restraint rates are now understood as evidence of treatment failure."[2] Since the Joint Commission began tracking sentinel events in 1996, it has reviewed the deaths of 20 patients who were physically restrained.[3] Since then, the Joint Commission has advocated standards based on prevention as an intervention and the use of restraint as a last resort only after the least restrictive measures are exhausted.

Most communities have a protocol to call for team assistance when a psychiatric patient begins to display aggression or when an ordinarily calm individual becomes agitated while on excitatory drugs such as methamphetamine, cocaine, or phencyclidine. Law enforcement often believe that there is power in numbers, which can be true in certain situations. However, the increased external stimuli of gathering more police officers can also have untoward effects on the patient. The show of force may contribute to the escalation of combative behaviors.

Evidence points to a direct correlation between a high level of anxiety or perceived powerlessness on the patient's part and ensuing aggression.[4] The underlying cause of the behavior should be readily identified and handled accordingly. For instance, patients can become angry as a result of hallucinations, external provocation, or physical discomfort.

The Third-Person Approach

Although restraint may be necessary in emergency situations for patient and first responder safety, physical confrontation can usually be averted if de-escalation techniques are implemented before the patient gets out of control. De-escalation using a third-person approach, if implemented judiciously and cautiously by first responders, can be very effective in managing patients in the early stages of anger and aggression.

The third-person approach is similar to hostage crisis negotiation, in which a third party is brought in to negotiate a solution. Usually, it is much easier for the third person to take a neutral stance and to allow space for the angry person to step down. Billikopf postulates that all other things being equal, an outside third party has a greater chance than an insider of successfully mediating and resolving a difference.[6] The third person is not an arbiter trying to decide right from wrong, but a nonjudgmental facilitator of communication.

A "third party" or "third person" is a trained crisis interventionalist who was not present at the start of the dispute or conflict. A person who was involved in the conflict may be perceived, from the patient's standpoint, as being part of the problem. The ideal third person is someone who knows the patient well and with whom the patient has a certain degree of rapport.

The value of a therapeutic relationship has been a known and established fact for many decades. Research suggests that ineffective interpersonal relationships and interactions are major factors in escalating the aggressive behavior of a volatile individual.[8,9] Irwin concludes that intolerable environments and ineffectual interactions are far more likely to influence behaviors than are psychiatric symptoms alone.[10]

Use of the Third Person in De-escalation

Whenever an outburst is anticipated, the audience should be removed immediately. If team assistance is called in accordance with institutional policy, it may be better for the team members to stay in the background, ready to provide support when needed, but allow a single, third person from the care team to approach the patient. This less-than-expected response, or "under-reaction," can promote de-escalation.[11] The Pennsylvania Patient Safety Authority also suggests shifting the method of intervention from "a show of force to a show of support."[12] A 3-month study on the use of least restrictive interventions found that patients commonly select "verbal warning or talking things through" as the most valuable tool of anger management.[13] In short, the men (and women) in blue need to put their guns away and back off.  Crisis intervention teams will have greater success at de-escalating any situation, and there will be much less risk of injury to the patient and to law enforcement.

The third person should maintain a calm and supportive demeanor and use therapeutic communication skills. Avoid arguing with the patient or getting into a power struggle, and listen with empathy; the Greek Stoic philosopher Epictetus said that we have 2 ears and 1 mouth, so that we can speak less and listen more. In addition, state everything in clear, simple language: As anger escalates, the patient's perceptual field becomes limited; he or she probably cannot understand complex reasoning or process what you are saying. Tell the patient that you want to help, but he or she needs to calm down first. It is appropriate to say something like, "I would like to help you, but I can't hear you if you are screaming and yelling." Do not react to verbal attacks from the patient. Be aware of your own feelings of countertransference.

Staff members who take on the role of third person should have proper training in various techniques of nonviolent crisis intervention. The third person must also practice safety precautions, such as standing beyond arm's reach of the patient, positioning himself or herself for easy escape, and avoiding displays of body language that may be viewed as provocative to the patient.

Sometimes patients act out because they feel threatened. Assure the patient that he or she is safe, then set firm but nonthreatening limits. Offer choices to gain the patient's cooperation, and present positive reinforcement first. Positive reinforcement does not have to be a material reward; it can be praise and encouragement, or earning a certain privilege. In Rosenheck and Neale's 6-month study of 40 Veterans Affairs Assertive Community Treatment Program teams, clients with violent behavior who were exposed to negative limit-setting interventions typically had poorer outcomes.[14]

First responders have an obligation to maintain the safety of the patient and others in the environment. If restraint is deemed necessary, it should be used only when all measures of de-escalation have failed. In reality, no rigid policy or clinical guideline can spell out each and every scenario when physical restraint is the lesser of 2 evils. Crisis intervention workers have to rely on their own clinical judgment to weigh the risks and benefits of the measures they are considering. When to initiate physical restraint is a situation that depends on circumstances.[15]

References:

1. Weiss EM. Deadly restraint: a Hartford Courant investigative report. Hartford Courant. 1998;October 11-15. Section A.1 Available at" http://courant.ctnow.com/projects/restraint/data.stm

 

2. Stefan S. Legal and regulatory aspects of seclusion and restraint in mental health settings. Special edition: Violence and Coercion in Mental Health Settings: Eliminating the Use of Seclusion and Restraint. NTAC Networks. 2002;Summmer/Fall

 

3. Massachusetts Coalition for the Prevention of Medical Errors. Principles and Best Practice Recommendations to Improve Patient Safety Related to Restraint and Seclusion Use. 2003. Available at: http://www.macoalition.org/documents/RestraintPrinBestPrac-Definitions-Final.pdf Accessed on December 24, 2009.

 

4. Johnson B, Martin M, Guha M, Montgomery P. The experience of thought-disordered individuals preceding an aggressive incident. J Psychiatr Ment Health Nurs. 1997;4:213-220.

 

5. Davidson SE. The management of violence in general psychiatry. Advances in Psychiatric Treatment. 2005;11:362-370.

 

6. Billikopf G. Chapter 4: Interpersonal negotiation skills. In: Party-Directed Mediation: Helping Others Resolve Differences. Modesto, California: Regents of the University of California; 2009:85.

 

7. Peplau HE. Interpersonal Relations in Nursing. A Conceptual Frame of Reference for Psychodynamic Nursing. London: Macmillan; 1998.

 

8. Breeze J, Repper J. Struggling for control: the care experiences of “difficult” patients in mental health services. J Adv Nurs. 1998;28:1301-1311.

 

9. Secker J, Benson A, Balfe E, Lipsedge M, Robinson S, Walker J. Understanding the social context of violent and aggressive incidents on an inpatient unit. J Psychiatr Ment Health Nurs. 2004;11:172-178.

 

10. Irwin A. The nurse's role in the management of aggression. J Psychiatr Ment Health Nurs 2006;13:309-318.

 

11. Kreisberg L. Constructive Conflicts. Lanham, Md: Rowman & Littlefield; 1998:181-222

 

12. Pennsylvania Patient Safety Authority. The PA-PSRS Patient Safety Advisory. 2005; 2:1-6. Available at: http://patientsafetyauthority.org/ADVISORIES/AdvisoryLibrary/2005/Mar2(1)/Documents/22.pdf Accessed December 24, 2009.

 

13. Morales E, Duphorne PL. Least restrictive measures: alternatives to four-points and seclusion. J Psychosoc Nurs Ment Health Serv. 1995:33:13-16.

 

14. Rosenheck RA, Neale MS. Therapeutic limit setting and six-month outcomes in a Veterans Affairs assertive community treatment program. Psychiatric Serv. 2004;55:139-144.

 

15. Mohr WK, Petti TA, Mohr BD. Adverse effects associated with physical restraint. Can J Psychiatry. 2003:48:330-337.

EMTALA Violations, ER Overcrowding, and Litigation

The following excerpt on EMTALA (Emergency Medical Treatment and Active Labor Act) is taken from a paper written by Dr. Damon Dietrich and Dr. Michael Crapanzano. The paper is entitled, "Emergency Department Diversion and Overcrowding: A Public Health Crisis." The paper discusses EMTALA in the context of ED diversion (EDD) and ED overcrowding (EDO).

In the U.S. today with increasing ER patient burden EMTALA violations have increased. This has resulted in increased litigation as well.

EDO and EDD are two of the most critical public health issues facing our nation's healthcare system today.[3,4,5]

  • ED visits in 2003 rose to 114 million, up from 97 million in 1997.
  • While visits increased by 17% over six years, 1128 EDs closed between 1988 and 1998, thus resulting in a dramatic increase in patient volumes and waiting times.
  • US hospitals over the past ten years have closed more than 103,000 inpatient medical/surgical beds and 7800 ICU beds in an effort to control costs.
  • The majority of the nation's 4000 hospitals and EDs report operating at or above critical capacity.
  • In 2001, two out of every three hospitals reported diverting ambulances to other hospitals.
  • EDO is reported to be most severe in areas with larger populations, higher population growth and higher than average uninsured patient volumes.

EDs represent the most critical access path to the nation's health delivery system, as the "guaranteed access point for all who need care regardless of ability to pay."[2] EDO exists when the ED has more patients than bed capacity or is over-saturated; this is a warning sign of capacity constraints under normal conditions. The March 2003 General Accounting Office Report indicated that EDO has many negative implications with regard to quality of care including prolonged patient wait times and suffering for acute problems while other patients are "boarded" in the ED, higher physician and staff stress, less confidentiality when patients are evaluated in nontraditional locations such as a hallway or on the EMS stretcher and increased transport times for ambulance patients due to diversion.[5] Contrary to public misconception, the gridlock in the ED is not in the waiting room, but rather occurs in the hallways of the ED with admitted patients waiting for beds upstairs for hours to days! The purpose of EDD was an innovative solution for EDO intended to "divert" stable patients transported by ambulance away from the hospital, thereby allowing the scarce beds remaining in the ED to be used for critical or unstable patients. EDD has been defined by The Lewin Group.[2]

 

Read more: EMTALA Violations, ER Overcrowding, and Litigation

What happened to the SHAVING BLADE!

Background

Figure 1.
Figure 1.
(Click to enlarge)
Figure 2.
Figure 2.
(Click to enlarge)

A 41-year-old cognitively impaired man who lives in a home for the mentally impaired is presented to the emergency department (ED) by his caregivers. His caregivers at the home noticed that, after washing the patient, 1 of 2 shaving blades present in the morning was missing. Although they hadn't witnessed it, they suspected that the patient had ingested the missing shaving blade. The caregivers, however, haven't noticed anything abnormal with the patient; he does not have any shortness of breath, stridor, wheezing, hemoptysis, vomiting, or upper gastrointestinal (GI) bleeding. At no point have the caregivers noticed the patient coughing or choking; however, they are still worried about the possibility that he might have ingested or aspirated the blade.

At baseline, the patient has unintelligible speech and, therefore, cannot be directly interviewed. The patient has been living in a residence for the mentally impaired for the past 31 years, and he is dependent on his caregivers for all of the activities of daily living. Apart from his mental underdevelopment, the patient suffers from epilepsy and has been previously investigated for Raynaud's disease; in addition, the patient's history is significant for an ejection systolic murmur. The murmur was first detected by echocardiography, which revealed normal left ventricular function and a slightly thickened mitral valve, with minimal regurgitation. He takes multiple medications, including dipyridamole 100 mg TID, phenytoin 100 mg TID, doxepin 25 mg BID, nitrazepam 5 mg nocte, carbamazepine 20 mg BID, nifedipine 10 mg BID, nortriptyline 25 mg TID, and risperidone 1 mg nocte.

On physical examination, the patient is not in any distress. His oral temperature is 98.6°F (37.0°C). His pulse is regular at 82 bpm and his blood pressure is 140/90 mm Hg. His oxygen saturation is 99% while breathing room air. A visual inspection of the oropharynx does not reveal the presence of a foreign body, and there is no evidence of any recent trauma, bleeding, or swelling. There is normal range of motion in the neck, and no evidence of stridor or respiratory difficulty is noted. On chest auscultation, there is good air entry bilaterally, and normal vesicular breathing is heard. His heart sounds reveal a 2/6 ejection systolic murmur. His abdomen is soft and nontender. Additionally, no distention or rebound is noted on the abdominal examination.

Read more: What happened to the SHAVING BLADE!

Woman with Sudden Onset Nausea and Vomiting: Could this be a case for a Toxicology Expert Witness?

Background
Figure 1.
Figure 1.
(Click to enlarge)

A 46-year-old woman presents to the emergency department (ED) with a history of worsening, constant right upper quadrant pain that radiates to her back and side. She has had nausea and has vomited twice in the past several hours. The intake triage nurse considers that she may have ingested something toxic, and turns immediately to her Emergency Physician who is also a well known toxicology expert consultant.  But he learns upon questioning that she underwent a laparoscopic cholecystectomy 2 weeks ago, without complications, and returned to her normal diet. She has not had any bowel movements or flatulence since the pain began. She denies having any fever, chills, or rigors. Her medical history is significant only for high blood pressure, high cholesterol levels, and gallbladder disease. She takes lisinopril, aspirin, multivitamins, and ginseng. She denies smoking or drinking alcohol.

On physical examination, the patient is awake, alert, and oriented. Her vital signs are within the normal range, with a heart rate of 84 bpm and a blood pressure of 124/76 mm Hg. She appears to be in mild distress. The cardiorespiratory examination yields normal findings, with clear lungs and a regular heart rhythm. Her abdomen is soft, but her bowel sounds are decreased, and she has marked tenderness in the right upper quadrant. The rest of her abdomen is minimally tender, with no evidence of guarding or rebound and no palpable masses. The other physical findings are normal.

The laboratory investigation reveals an elevated WBC count of 14.0 × 109/L (14.0 × 103/µL), with a left shift of 87% neutrophils. Her liver function tests, lipase level, and basic chemistry panel are unremarkable.

Contrast-enhanced computed tomography (CT) scans of the abdomen and pelvis are ordered. Figure 1A shows an anteroposterior (AP) scout image, and Figure 1B shows a selected axial section. The appearance of the xrays is that of bean-like substances.

The diagnosis is Cecal Volvulus.

 

Read more: Woman with Sudden Onset Nausea and Vomiting: Could this be a case for a Toxicology Expert Witness?

Bird Droppings and a Chronic Cough

A 46-year-old man presents to the emergency department (ED) with a 4-day history of dry cough, fever, chills, night sweats, a 13-lb (5.9-kg) weight loss, (over a 2-week period), shortness of breath, and easy fatigability. Two weeks before presentation, he went with a friend to clean an abandoned house in Kentucky. The house was very dusty and had a lot of bird droppings. His friend developed a fever with cough and could not continue working after 4 days. Our patient, however, continued to work until the job was done (a total of 7 days). He has not had any hemoptysis, chest pain, or any other symptoms, except as noted above. He has mild asthma, uses an albuterol inhaler once every 3-4 months, has smoked half of a pack of cigarettes per day for the last 30 years, does not drink alcohol, and does not use any illicit drugs. He has no significant family history and is not allergic to any medications.

On physical examination, his oral temperature is 98.9°F (37.2°C). His pulse is regular, with a rate of 98 bpm, and his blood pressure is 137/91 mm Hg. He is mildly tachypneic, with a respiratory rate of 22 breaths/min. The patient is thin but not emaciated, and he has poor dentition; his head and neck examination is otherwise normal, with no palpable lymph nodes. He has normal S1 and S2 heart sounds. Chest auscultation reveals fine rales in the mid and lower zones of both lungs. His abdomen is soft and nontender, with normal bowel sounds. He does not have any lower limb edema, and he has normal peripheral pulses.

Read more: Bird Droppings and a Chronic Cough

Call Emergency Medicine Expert 510-549-1041
Copyright © 2020 - toxicologyexpert.net